Question Summary
0 of 105 questions completed
Questions:
- 1
- 2
- 3
- 4
- 5
- 6
- 7
- 8
- 9
- 10
- 11
- 12
- 13
- 14
- 15
- 16
- 17
- 18
- 19
- 20
- 21
- 22
- 23
- 24
- 25
- 26
- 27
- 28
- 29
- 30
- 31
- 32
- 33
- 34
- 35
- 36
- 37
- 38
- 39
- 40
- 41
- 42
- 43
- 44
- 45
- 46
- 47
- 48
- 49
- 50
- 51
- 52
- 53
- 54
- 55
- 56
- 57
- 58
- 59
- 60
- 61
- 62
- 63
- 64
- 65
- 66
- 67
- 68
- 69
- 70
- 71
- 72
- 73
- 74
- 75
- 76
- 77
- 78
- 79
- 80
- 81
- 82
- 83
- 84
- 85
- 86
- 87
- 88
- 89
- 90
- 91
- 92
- 93
- 94
- 95
- 96
- 97
- 98
- 99
- 100
- 101
- 102
- 103
- 104
- 105
Information
Start exam
You have already completed the Exam before. Hence you can not start it again.
Exam is loading...
You must sign in or sign up to start the Exam.
You have to finish following Exam, to start this Exam:
Results
0 of 105 questions answered correctly
Your time:
Time has elapsed
You have reached 0 of 0 points, (0)
Categories
- Not categorized 0%
- 1
- 2
- 3
- 4
- 5
- 6
- 7
- 8
- 9
- 10
- 11
- 12
- 13
- 14
- 15
- 16
- 17
- 18
- 19
- 20
- 21
- 22
- 23
- 24
- 25
- 26
- 27
- 28
- 29
- 30
- 31
- 32
- 33
- 34
- 35
- 36
- 37
- 38
- 39
- 40
- 41
- 42
- 43
- 44
- 45
- 46
- 47
- 48
- 49
- 50
- 51
- 52
- 53
- 54
- 55
- 56
- 57
- 58
- 59
- 60
- 61
- 62
- 63
- 64
- 65
- 66
- 67
- 68
- 69
- 70
- 71
- 72
- 73
- 74
- 75
- 76
- 77
- 78
- 79
- 80
- 81
- 82
- 83
- 84
- 85
- 86
- 87
- 88
- 89
- 90
- 91
- 92
- 93
- 94
- 95
- 96
- 97
- 98
- 99
- 100
- 101
- 102
- 103
- 104
- 105
- Unanswered
- Answered
- Review
-
Question 1 of 105
1. Question
Whilst hospitalized after a hip replacement, a patient with acute intermittent porphyria has a seizure thought to be due to alcohol withdrawal. Patient requires large amounts of analgesia. Six days post-operatively, she suffers a porphyric attack. Which of the following drugs is most likely responsible?
Correct
Sodium valproate
Incorrect
Explanation:
AIP is an autosomal dominant disease resulting from defects in production or activity of the enzyme porphobilinogen (PBG) deaminase. Attacks may be precipitated by fasting, and by many drugs, such as phenobarbitone, alcohol, sulphonylureas and oestrogens. Amoxicillin and opiates are thought to be safe in porphyria.
-
Question 2 of 105
2. Question
A 20 year old man falls onto his outstretched left hand. On examination, there is pain on deep palpation at the base of the thumb.
Which one of the following bones is most likely fractured?Correct
Scaphoid
Incorrect
Explanation:
A fall on an outstretched arm often results in a fracture of the scaphoid (navicular) bone in the wrist. This small bone is one of 8 carpal bones in the wrist. The scaphoid sits below the thumb, and is shaped like a kidney bean. This complex bone has a unique and limited blood supply that can be easily disrupted by a fracture.
If there is pain or deep aching on the thumb-side of the wrist, typically after a fall on an outstretched arm, the patient likely has a scaphoid fracture.
Other symptoms include swelling in the wrist, difficulty gripping objects, and tenderness or pain in the anatomical snuff box (a sunken space between tendons of the wrist). Many patients are diagnosed with a wrist sprain, when they actually have a fracture. Diagnosis is difficult because the fracture often doesn´t appear on x-rays until weeks later, after healing has begun. Because of this, it is common for physicians to treat a wrist injury as though it were a scaphoid fracture initially, and then repeat x-rays within two weeks. -
Question 3 of 105
3. Question
A 25 year old man comes to the emergency department because of pain, swelling and erythema over the metacarpophalangeal joint of the right long finger. Three days ago he struck an opponent in the mouth during an altercation. On physical examination there is an abrasion over the dorsal surface of the joint, pain on passive motion of the finger, and tenderness along the volar aspect of the finger and metacarpal. His temperature is 38.5°C (101.3°F). X-ray films are normal. Which of the following is the most appropriate management?
Correct
Surgical debridement and intravenous antibiotic therapy
Incorrect
Explanation:
The history and physical findings suggest an infective arthritis of the metacarpophalangeal joint. This is most commonly from a puncture or cut on the dorsum of the hand; it is often seen when a fighter´s fist strikes the tooth of his opponent, resulting not only in crushed tissue, but also in inoculation of organisms in the metacarpophalangeal joint, particularly into the metacarpal head. Wounds of human bites should be debrided and irrigated, not sutured, and aggressive intravenous antibiotic therapy is immediately started. All of the other choices, including splinting and exercise, are usually indicated in fracture and dislocation of phalangeal and metacarpal bones. Splinting, antibiotics and reevaluation and splinting and antibiotic therapy are incorrect because he requires debridement.
-
Question 4 of 105
4. Question
A 26 year old male comes to your office complaining of pain in the right wrist since falling 2 weeks ago. On examination he is tender in the anatomic snuffbox. A fracture of the proximal one-third of the scaphoid is visualized on the radiograph. Which one of the following is the most appropriate management at this time?
Correct
A thumb spica cast
Incorrect
Explanation:
Fracture of the scaphoid should be suspected in every “sprained wrist´ where there is tenderness in the anatomic snuffbox. Radiographs may be negative initially. The scaphoid circulation enters the bone for the most part through the distal half. Fractures though the proximal third tend to cause loss of circulation and are slower to heal, and should be referred to an orthopaedist because of the risk of nonunion and avascular necrosis. Fractures through the middle of distal one-third can be handled by the family physician in consultation with an orthopaedist. The fracture is treated with a thumb spica cast for 10-12 weeks. A wrist splint does not provide adequate immobilization. A bone scan is unnecessary, and physical therapy is inappropriate. If there is still no evidence of union after 10 weeks of immobilization, the patient should be referred to an orthopaedist for further care.
-
Question 5 of 105
5. Question
A 12 year old boy is brought to your office after sustaining a fall on an outstretched hand. Radiographs show a nondisplaced fracture of the middle third of the clavicle. Appropriate management would include which one of the following?
Correct
Intermittent heat to the area to help control pain
Incorrect
Explanation:
In treating the midshaft clavicular fracture, the goal is reduction of motion at the fracture site. This rarely requires operative intervention and can be managed by the family physician without orthopedic referral. The fracture site is best stabilized by restricting shoulder motion to less than 45° abduction. Either an arm sling or a figure-of-8 clavicular splint holding the shoulder back at ´the “position of attention´ may be used. The figure-of-8 splint offers the advantage of leaving the elbow and hand free for activity. Ice and analgesics are used as needed during the acute stage of injury. Early use of heat may increase the inflammatory response. The patient may use the arm as the pain permits, but should not abduct the arm more than 45° for several weeks. The risk of adhesive capsulitis is negligible in children. Repeat radiographs at each follow-up office visit are not necessary, but a final radiograph should be ordered when clinical union has occurred to assess callus formation.
-
Question 6 of 105
6. Question
A.72 year old white female is scheduled to undergo a total knee replacement for symptomatic osteoarthritis. She is otherwise healthy, with no history of vascular disease. Which one of the following is most appropriate for prophylaxis against deep vein thrombosis?
Correct
Enoxaparin (Lovenox), 30 mg subcutaneously every 12 hours
Incorrect
Explanation:
Prophylaxis is indicated with total knee or hip replacements. The two regimens recommended are low-molecular-weight heparin and adjusted-dose warfarin. These may be augmented by intermittent pneumatic compression.
-
Question 7 of 105
7. Question
A 14 year old boy fractures his leg during a football match. After a few days in a cast he develops pain on passive extension. What is the most likely cause?
Correct
Compartment syndrome
Incorrect
Explanation:
Compartment syndrome is increased tissue pressure within a closed fascial space, resulting in tissue ischemia. The earliest symptom is pain out of proportion to the severity of injury. Common causes include fractures and severe contusions. Compartment syndrome usually occurs in the anterior lower leg. The earliest symptom is worsening pain. It is typically out of proportion to the severity of the apparent injury and is exacerbated by passive stretching of the muscles within the compartment. Pain, one of the 5 P´s of tissue ischemia, is followed by the other 4: paresthesias, paralysis, pallor, and pulselessness.
Diagnosis is by measuring compartmental pressure (normal = 20 m Hg), usually with a commercially available wick catheter. Pressures of 20 to 40 mm Hg can sometimes be treated conservatively with analgesics, elevation, and splinting. Casts, if present, are removed or bivalved. Pressures > 40 mm Hg usually require immediate fasciotomy to relieve pressure. If necrosis occurs, amputation may be needed. -
Question 8 of 105
8. Question
A 26 year old man has a week´s history of knee pain. He says he jogs 3 miles a day and that the pain in his knee worsens during his run. His gait appears to be normal on physical examination. Examination of the right knee reveals tenderness and fullness over the medial collateral ligament. On physical examination there is most likely to be
Correct
Pain during forceful abduction of the leg
Incorrect
Explanation:
This is a classic description of injured medial collateral ligaments, which results from constant stress in valgus position. Integrity of the medial collateral ligament is tested by performing forceful abduction of the leg while placing one hand on the lateral aspect of the knee, which is flexed about 20 degrees, and pushing the leg outward with the right hand. Abduction with the opening of the joint line more than 5 mm and associated pain usually indicates rupture of the medial collateral ligament. -
Question 9 of 105
9. Question
A 19 year old motorcyclist presents in the emergency department following an accident. Radiographic investigation reveals a compound tibia and fibula fracture of the right leg and on examination the right leg has no pulses. Your immediate treatment should be
Correct
Reduction and splinting
Incorrect
Explanation:
Immediate treatment of tibia and fibula fractures includes analgesics and, for suspected unstable fractures or fractures of long bones, splinting. Suspected open fractures require sterile wound dressings, tetanus prophylaxis, and broad-spectrum antibiotics (eg, a 2nd-generation cephalosporin plus an aminoglycoside). Rotational malalignment or significant angulation or displacement is corrected with reduction (realignment of bone fragments by manipulation). Exceptions include some diaphyseal fractures in children. In these fractures, remodeling gradually corrects some types of significant angulation, and end-to-end realignment of fractured bone fragments can stimulate bone growth, which may then be excessive. Closed reduction (without skin incision) is done when possible; if not, open reduction (with skin incision) is done. In open reduction and internal fixation (ORIF), fracture fragments are aligned and held in place using hardware.
-
Question 10 of 105
10. Question
A 26 year old player severely fractures his ankle during a game of football and subsequently requires ORIF treatment. Indications for ORIF treatment of an ankle fracture include all of the following, except
Correct
Unstable talar tilt
Incorrect
Explanation:
Closed reduction (without skin incision) is done when possible; if not, open reduction (with skin incision) is done. In open reduction and internal fixation (ORIF), fracture fragments are aligned and held in place using hardware. ORIF is usually indicated for the following:
- When an intra-articular fracture is displaced (to precisely align the joint cartilage).
- When ORIF has been shown to have better results for a particular type of fracture.
- When closed reduction was ineffective.
- When the fracture traverses a cancerous lesion (because normal bone healing does not occur).
- When prolonged immobility (required for callus formation and remodeling) is undesirable (eg, for hip fractures), because ORIF provides early structural stability, which facilitates mobilization.
-
Question 11 of 105
11. Question
Which one of the following is consistent with spinal stenosis but not herniated vertebral disk?
Correct
Pain relieved by sitting
Incorrect
Explanation:
Causes of low back pain include vertebral disk herniation and spinal stenosis. Numbness and muscle weakness may be present in both pain in spinal stenosis is relieved by sitting and aggravated by standing. Pain from a herniated disk is aggravated by sitting and relieved by standing.
-
Question 12 of 105
12. Question
Which one of the following is the most serious complication of a slipped capital femoral epiphysis is
Correct
Avascular necrosis
Incorrect
Explanation:
Avascular necrosis is the most serious complication of a slipped capital femoral epiphysis, and leads to more rapid arthritic deterioration. It may require hip fusion and total hip replacement early in adulthood.
-
Question 13 of 105
13. Question
Two hours after application of the plaster cast for supracondylar fracture, a patient comes back to emergency room with a complaint of severe pain in the hand. On examination it is revealed that fingers are swollen and cyanosed. Which one of the following is the best management?
Correct
Cut open the entire plaster cast immediately
Incorrect
Explanation:
Volkmann´s ischemia occurs when there is a lack of blood flow to the forearm, usually caused by increased pressure that results from swelling (compartment syndrome).
The hallmark symptom is pain that does not improve with rest or non- sedating pain medications, and will continue to get worse with time. If the pressure is allowed to persist, there will be decreased sensation, weakness, and paleness of the skin.
The best treatment is early surgery to release the pressure in the forearm before any permanent injury to the muscles and nerves occurs. -
Question 14 of 105
14. Question
A 22 year man who is a competitive runner has pain when walking. The pain has increased recently and is noted in his 2nd and 3rd metatarsal bone of his forefoot. What is the most likely diagnosis?
Correct
Stress fracture
Incorrect
Explanation:
Metatarsal stress fractures involve the metatarsal shafts and are caused by repetitive weight-bearing stress.
Stress fractures do not result from a discrete injury (eg, fall, blow) but occur after repeated stress. Metatarsal stress fractures (march fractures) usually occur in runners and in poorly conditioned patients who walk long distances carrying a load (eg, new recruit soldiers); they most commonly occur in the 2nd metatarsal. Risk factors include a cavus foot (high arch), shoes with inadequate shock-absorbing qualities, and osteoporosis; these fractures also may be a sign of the female athlete triad (amenorrhea, eating disorder, and osteoporosis).
Forefoot pain that occurs after a long or intense workout, then disappears shortly after stopping exercise is the typical initial presentation. With subsequent exercise, onset of pain is progressively earlier, and pain may become so severe that it prohibits exercise and persists even when the patient is not bearing weight. -
Question 15 of 105
15. Question
A 23 year hockey player gets hit on his face with a puck traveling at high speed. He presents with facial pain, diplopia and malocclusion of his teeth. Which of the following fractures is most likely to have occurred?
Correct
Maxillary
Incorrect
Explanation:
Many situations can cause facial fractures. Motor vehicle crashes, sporting injuries, falls, and assault account for the majority, although injuries from gunshot wounds and stabbings occur as well.
Always be concerned about other injuries with facial fractures. In particular, other parts of the body may be injured, for example, if a patient has facial injuries in a motor vehicle accident.
In a midface (maxillary) fracture symptoms include inability to bring the teeth together properly, visual problems and clear nasal discharge. Bruising may be present around the eyes and the midface may be able to be moved.
These fractures are not usually subtle and are often the result of high- speed car accidents. As a result, there might be severe injury to areas other than the face. -
Question 16 of 105
16. Question
A woman aged 38 presents with pain on the lateral aspect of her elbow. Her pain is reproduced by extending the wrist against resistance with the elbow extended. The most likely diagnosis is?
Correct
Lateral epicondylitis
Incorrect
Explanation:
Lateral epicondylitis (tennis elbow) is a degenerative condition of the tendon fibers that attach on the bony prominence (epicondyle) on the outside (lateral side) of the elbow. The tendons involved are responsible for anchoring the muscles that extend or lift the wrist and hand.
Patients often complain of severe, burning pain on the outside part of the elbow. In most cases, the pain starts in a mild and slow fashion. It gradually worsens over weeks or months.
The pain can be made worse by pressing on the outside part of the elbow or by gripping or lifting objects. The doctor will perform a physical examination. The doctor may press directly on the bony prominence on the outside part of the elbow to see if it causes pain. The doctor may also ask the patient to lift the wrist or fingers against pressure to see if that causes pain. -
Question 17 of 105
17. Question
A woman aged 52, who wears high-heeled, pointed shoes, complains of pain in the forefoot after prolonged standing or walking. Occasionally, she also experiences numbness, tingling and burning sensation in the area. A very tender spot in the third interspace, between the third and fourth toes is present. There is no redness or signs of inflammation. What is your likely diagnosis?
Correct
Morton´s neuroma
Incorrect
Explanation:
The location and circumstances are classic for Morton´s neuroma, a benign neuroma of the third plantar interdigital nerve. Gout happens to obese, elderly males, and redness and signs of inflammation in the affected joint are evident. Hallux rigidus is osteoarthritis of the first metatarsophalangeal joint. There is deformity and limitation of motion. The joint is tender on physical examination. Metatarsophalangeal articulation pain is likewise associated with misalignment of joint surfaces. There is pain when examining the joint, and there is no history of numbness, ´burning, or tingling. Plantar fasciitis produces sharp pain on physical examination when pressing the plantar surface of the heel.
-
Question 18 of 105
18. Question
An older, overweight man complains of disabling, sharp heel pain every time his foot strikes the ground. The pain is worse in the mornings. A bony spur is found matching the location of his pain, and exquisite tenderness is present to direct palpation right over that heel spur. Furthermore, when the ankle is dorsiflexed, the entire inner border of the fascia is tender to palpation. Which of the following is the most likely diagnosis?
Correct
Plantar fasciitis
Incorrect
Explanation:
All the details are in the vignette, including the association with a heel spur that in the past led many of these patients to undergo-unnecessary surgery to remove the spur. The spur is caused by the pull of the fascia and is not the cause of the plantar fasciitis. Epiphysitis of the calcaneus affects children, and the pain occurs along the sides of the heel where the heel growth centers are located.
-
Question 19 of 105
19. Question
A football player of age 25 years sustains a finger injury during practice. According to him his finger got caught in another team member´s jersey. His right ring finger is tender on the volar surface, and is swollen around the distal interphalangeal joint. He is unable to flex at the distal interphalangeal joint. No bone abnormality is found on radiograph. You would now
Correct
Splint his finger in a slightly flexed position and urgently refer him to a hand surgeon
Incorrect
Explanation:
This injury is commonly called jersey finger. It is a tendinous rupture, sometimes with a bony avulsion fracture of the flexor digitorum profundus tendon. The tendon usually retracts, which makes nonsurgical treatment unlikely to result in complete healing. Splinting in hyperextension will distract the two ends of the tendon and it will not heal. A cast or flexion splint is unlikely to lend itself to complete healing because the tendon ends are unlikely to reattach to each other.
-
Question 20 of 105
20. Question
The incidence of osteonecrosis of femoral head is high in
Correct
Femoral neck fractures
Incorrect
Explanation:
Femoral neck fractures disrupt the blood supply to the femoral head, which can lead to osteonecrosis. This does not occur with the other types of fractures.
-
Question 21 of 105
21. Question
A 72 year old white female with osteoporosis comes for a follow-up a few days after an emergency department visit for an acute T12 vertebral compression fracture. The emergency department physician prescribed oxycodone (OxyContin) and NSAIDs, but considerable discomfort is still present. In addition to increasing the dosage of oxycodone, which one of the following intervention would you suggest to decrease the pain?
Correct
Calcitonin (Miacalcin)
Incorrect
Explanation:
Calcitonin, either intranasal or subcutaneous, provides pain relief within a few days in many patients with osteoporotic vertebral compression fractures. The remainder of the choices does not provide acute pain relief. Vertebroplasty/kyphoplasty procedures are generally reserved for cases in which medical management has failed.
-
Question 22 of 105
22. Question
The most serious complication of supracondylar fracture of the humerus is?
Correct
A compartment syndrome of forearm
Incorrect
Explanation:
With supracondylar fractures, casting these injuries as means of definitive treatment carries risk of compartment syndrome. Compartment syndrome is increased tissue pressure within a closed fascial space, resulting in tissue ischemia.
Compartment syndrome is a self-perpetuating cascade of events. It begins with the tissue edema that normally occurs after injury (eg, because of soft-tissue swelling or a hematoma). If edema develops within a closed fascial compartment, typically in the anterior or posterior compartments of the leg, there is little room for tissue expansion, so interstitial (compartmental) pressure increases. As compartmental pressure exceeds about 20 mm Hg, cellular perfusion slows and may ultimately stop. Resultant tissue ischemia further worsens edema in a vicious circle. As ischemia progresses, muscles necrose, sometimes leading to rhabdomyolysis and infections; these complications can cause loss of limb and, if untreated, death.
Common causes include fractures and severe contusions. The earliest symptom is pain out of proportion to the severity of injury. Diagnosis is by measuring compartmental pressure. Treatment is fasciotomy. -
Question 23 of 105
23. Question
A 26 year old man suffers a severe car accident. Following accident the man was found to have an unstable knee joint. On examination under anesthesia, it is possible to pull the tibia forward excessively on the femur. The structure most likely injured in this patient is which one of the following?
Correct
Anterior cruciate ligament
Incorrect
Explanation:
Sprains of the external (medial and lateral collateral) or internal (anterior and posterior cruciate) ligaments or injuries of the menisci commonly result from knee trauma. Symptoms include pain, effusion, instability (with severe sprains), and locking (with some meniscal injuries). Diagnosis is by physical examination and sometimes MRI or arthroscopy.
The lachman´s test (anterior drawer test) is the most sensitive physical test for acute anterior cruciate ligament tears. With the patient supine, the examiner supports the patient´s thigh and calf, and the patient´s knee is flexed 20°. The lower leg is moved anteriorly. Excessive passive anterior motion of the lower leg from the femur suggests a significant tear. -
Question 24 of 105
24. Question
What is the most serious complication of a displaced supracondylar fracture of the humerus?
Correct
Compartment syndrome of the forearm
Incorrect
Explanation:
Compartment syndrome is increased tissue pressure within a closed fascial space, resulting in tissue ischemia. Common causes include fractures and severe contusions. The earliest symptom is pain out of proportion to the severity of injury. Diagnosis is by measuring compartmental pressure. Treatment is fasciotomy.
-
Question 25 of 105
25. Question
Which one of the following is false regarding knee injuries?
Correct
Lateral meniscus tears are more common than medial meniscal tears
Incorrect
Explanation:
The most commonly injured knee structures are the medial collateral and anterior cruciate ligaments. The most common mechanism for ligamentous knee injuries is an inward, medial force usually accompanied by some external rotation and flexion (as when being tackled in football). In such cases, the medial collateral ligament is usually injured first, followed by the anterior cruciate ligament, then the medial meniscus.
-
Question 26 of 105
26. Question
An 81 year old male falls while walking down stairs. He is brought to the emergency department with a 3 part intertrochanteric hip fracture. You choose to perform which of the following procedure?
Correct
Pin and plate
Incorrect
Explanation:
There are two common types of hip fractures. Femoral neck or subcapital hip fractures occur in the neck of the femur. Intertrochanteric fractures occur in the large bony bumps (trochanters) where the powerful muscles of the buttocks and legs attach. When the fracture is not too severe, metal pins can be inserted surgically to support the femoral head. This surgical procedure preserves the person´s own hip joint.
Most people with a hip fracture are treated with surgery. The type of surgery depends on the type of fracture. Treatment of severe femoral neck hip fractures involves removing the broken pieces surgically because the blood supply to the femoral head has been damaged. If damage to the femoral neck is incomplete (the break does not go all the way through), metal pins can be inserted surgically to support the femoral head (internal fixation). This is a smaller surgical procedure and the person´s own hip joint is preserved.
Intertrochanteric hip fractures are treated with an implant, such as a sliding compression screw and side plate. This implant securely holds the bone fragments in their proper position while the fracture heals. The fixation is usually strong enough to permit the person to bear weight as tolerated. While the bone fragments generally heal in a couple of months, most people continue to improve in terms of comfort, strength, and walking ability for at least 6 months. -
Question 27 of 105
27. Question
A 17 year old girl fell while roller-blading on her outstretched right hand. She is brought to the emergency department. X-rays confirmed the diagnosis of a closed Colles fracture. Which of the following is the proper reduction technique for this fracture?
Correct
Slight flexion, full pronation, and full ulnar deviation
Incorrect
Explanation:
Colles´ fracture is a common type of injury. In order to improve hand, wrist and elbow functions appropriate rehabilitation program should be applied soon after the orthopedic intervention.
The treatment procedure consists of closed reduction and the application of a below dorsal elbow cast immobilizing the wrist in 20° palmar flexion and in 20° ulnar deviation for 6 weeks. Rehabilitation program is of utmost importance in Colles´ fracture so it should be applied soon after the orthopedic manipulation. -
Question 28 of 105
28. Question
Out of following, which is true regarding mandibular fractures?
Correct
Malocclussion of teeth is a common sign
Incorrect
Explanation:
Fractures of the lower jaw (mandible) are suspected in patients with post-traumatic malocclusion or focal swelling and tenderness over a ´segment of the mandible. Other clues include defects of the dental occlusal surface, alveolar ridge disruptions, and anesthesia in the distribution of the inferior alveolar or mental nerve.
-
Question 29 of 105
29. Question
False statement with regard to falls in the elderly is
Correct
Fractures most commonly involve the humerus
Incorrect
Explanation:
Falls are the leading cause of injury related visits to emergency departments and the primary etiology of accidental deaths in persons over the age of 65 years. The mortality rate for falls increases dramatically with age in both sexes and in all racial and ethnic groups, with falls accounting for 70 percent of accidental deaths in persons 75 years of age and older. Falls can be markers of poor health and declining function, and they are often associated with significant morbidity.
More than 90 percent of hip fractures occur as a result of falls, with most of these fractures occurring in persons over 70 years of age. One third of community-dwelling elderly persons and 60 percent of nursing home residents fall each year. Risk factors for falls in the elderly include increasing age, medication use, cognitive impairment and sensory deficits. Outpatient evaluation of a patient who has fallen include: a focused history with an emphasis on medications, a directed physical examination and simple tests of postural control and overall physical function. -
Question 30 of 105
30. Question
A 33 year old female marathon runner complains of persistent pain and tenderness over the pubic symphysis. With exercise, pain is noted in the lower abdomen and medial thighs. Pain is not associated with fever. Widening of pubic symphysis is seen on a radiograph. Which of the following is most likely diagnosis?
Correct
Osteitis pubis
Incorrect
Explanation:
Osteitis pubis is characterized by the chronic occurrence of pain in the lower abdomen and medial thighs with exercise. It produces tenderness over the pubic symphysis, and widening of the symphysis is -often seen on plain radiographs. Pain from osteomyelitis is similar, but usually presents with fever and systemic symptoms, and does not cause widening of the symphysis. A stress fracture of the pubic ramus may occur in distance athletes, but the pain and tenderness is lateral, located over the posterior ramus. Inguinal hernia should not present with symphysis pain.
-
Question 31 of 105
31. Question
A 90 year old man has a history of painless ulcer on the dorsum of the proximal interphalangeal joint of the second toe for 6 months. Examination shows a hallus valgus and a rigid hammer toe of the second digit. Mild to moderate atrophic skin changes are present on his foot and the dorsal and posterior tibial pulses are absent. Which of the following is appropriate treatment?
Correct
Custom-made shoes to protect the hammer toe
Incorrect
Explanation:
The treatment of foot problems in the elderly is difficult because of systemic and local infirmities, the most limiting being the poor vascular status of the foot. Conservative, supportive, and palliative therapy replaces definitive reconstructive surgical therapy. Surgical correction of a hammer toe and bunionectomy would be disastrous in an elderly patient with a small ulcer and peripheral vascular disease. The correct approach to this patient is to prescribe custom-made shoes and a protective shield with a central apeture of foam rubber placed over the hammer toe. Metatarsal pads are not useful in the treatment of hallux valgus and a rigid hammer toe.
-
Question 32 of 105
32. Question
An elderly woman with osteoporosis falls on her outstretched hand. She presents with a deformed and painful wrist that looks like a dinner fork. Radiograph show a dorsally displaced, dorsally angulated fracture of the distal radius. Associated fracture of the ulnar styloid is also present. A neurologic examination is normal. The most appropriate management is which of the following?
Correct
Closed reduction and long arm cast
Incorrect
Explanation:
This is the famous Colles fracture, which typically can be reduced well with closed manipulation. Casting should immobilize both the wrist and the elbow, thus a long arm cast is needed. A short arm cast would not immobilize the elbow. Skeletal traction could correct the deformity, but at the cost of limiting future function of the hand. An intramedullary rod is usually reserved for fractures of the shaft of long bones, like the femur. Open reduction and internal fixation could indeed provide a very nice result, but it would be an unnecessarily expensive and intrusive way to do it.
-
Question 33 of 105
33. Question
All of the following are indications for repair of an orbital blow-out fractures, except
Correct
Orbital floor lesion < 0.7 cm
Incorrect
Explanation:
Sometimes the eye is struck in such a way that the force of the blow is ´received by the eyeball and not blocked by the strong bones around the eye (as when struck by a small object such as a golf ball). In this case, the pressure on the eyeball is transmitted to the walls of the orbit. This pressure can fracture the most fragile part of the orbit, which is typically the part underneath the eyeball (orbital floor). This is known as a blowout fracture. Fractures can also occur to the sides (walls) and roof of the orbit. Sometimes parts of an eye or the muscles attached to it are forced through the fractured bone. Blowout fractures sometimes cause double vision, a sunken eyeball, an eyeball that is stuck looking downward, a decreased sensitivity to touch and pain around the cheek and upper lip (caused by injury to the nerves below the orbit), or an accumulation of air in the tissues under the skin (subcutaneous emphysema). Nosebleeds, swelling of the eyelid, and the passage of blood from ruptured blood vessels into tissue under the skin (marked by a purple discoloration of the skin) may also occur. A doctor who suspects an orbital fracture performs a computed tomography (CT) scan, which shows any fractures, collections of blood, and displaced or trapped tissue. Surgical repair of the facial bones is usually necessary if a blowout fracture traps muscles or soft tissues of the orbit and causes double vision or nerve injury or makes the eyeball sunken or if symptoms do not go away in 2 weeks.
-
Question 34 of 105
34. Question
A 42 year old athlete complains of gradually worsening pain on the lateral aspect of his foot. He runs on asphalt, and has increased his mileage from 2 miles/day to 5 miles/day over the last 2 weeks. Palpation causes pain over the lateral 5th metatarsal. The pain is also developed when he jumps on the affected leg. When asking about his shoes he tells you that he brought them several years ago. The most likely diagnosis is
Correct
Stress fracture
Incorrect
Explanation:
Running injuries are primarily caused by overuse due to training errors. Runners should be instructed to increase their mileage gradually, in increments of 10% or less each week. A stress fracture causes localized tenderness and swelling in superficial bones. Pain is reproduced by jumping on the affected leg. Plantar fasciitis causes burning pain in the heel and there is tenderness of the plantar fascia where it inserts onto the medial tubercle o the calcaneus.
-
Question 35 of 105
35. Question
A 20 year old girl presents to the emergency department with a history persistent with a lateral ankle sprain that occurred 2 hours ago while she was playing softball. She has pain over the distal anterior talofibular ligament, but is able to bear weight. Mild swelling is present; mild black and blue discoloration and moderate tenderness to palpitation over the insertion of the anterior talofibular ligament, but the malleoli are nontender to palpitation. Which is true regarding the management of this case?
Correct
Early range-of-motion exercises should be initiated to maintain flexibility
Incorrect
Explanation:
This patient has an uncomplicated lateral ankle sprain and requires minimal intervention. The Ottawa ankle rules were developed to determine when radiographs are needed for ankle sprains. In summary, ankle radiographs should be done if the patient has pain at the medial or lateral malleolus and either bone tenderness at the back edge or tip of the lateral or medial malleolus, or an inability to bear weight immediately after the injury or in the emergency department, or both. If the patient complains of midfoot pain and/or bone tenderness at the base of the fifth metatarsal or navicular, or an inability to bear weight, radiographs should be ordered.
Sprains can be differentiated from major partial or complete ligamentous tears by anteroposterior, lateral, and 300 internal oblique (mortise view) radiographs. If the joint cleft between either malleolus and the talus is >4 mm, a major ligamentous tear is probable tress radiographs in forced inversion are sometimes helpful to demonstrate stability, but ankle instability can be present with a normal stress radiograph.
Grade I and II ankle sprains are best treated with RICE (rest, ice, compression, elevation) and an air splint for ambulation. NSAIDs are used for control of pain and inflammation. Heat should be applied. Early range-of-motion exercises should be initiated to maintain flexibility. Weight bearing is appropriate as tolerated and functional rehabilitation should be started when pain permits. Exercises on a balance board will help develop coordination. -
Question 36 of 105
36. Question
The highest risk for postoperative deep venous thrombosis is present in which one of the following procedures?
Correct
Total knee replacement
Incorrect
Explanation:
Neurosurgical procedures, particularly those with penetration of the brain or meninges, and orthopedic surgeries, especially those of the hip, have been linked with the highest incidence of venous thromboembolic events. The risk is due to immobilization, venous injury and stasis, and impairment of natural anticoagulants. For total knee replacement, hip fracture surgery, and total hip replacement, the prevalence of DVT is 40%-80%, and the prevalence of pulmonary embolism is 2%-30%. Other orthopedic procedures, such as elective spine procedures, have a much lower rate, approximately 5%. The prevalence of DVT after a coronary artery bypass graft is approximately 5%, after transurethral prostatectomy < 5%, and after abdominal hysterectomy approximately 16%.
-
Question 37 of 105
37. Question
A boy of age 21 year presents to you after a fall on an outstretched hand while skateboarding. He has pain at the anatomic snuffbox with no abrasion. No finding is revealed on radiographs. The most appropriate management is
Correct
A thumb spica splint and follow-up radiographs in 2 weeks
Incorrect
Explanation:
This is the classic presentation of a possible scaphoid fracture. This fracture is important to diagnose and treat appropriately because of a high rate of non-union. If radiographs are negative, the patient should be placed in a thumb spica splint and have repeat radiographs in 2 weeks, because initial studies may be negative. An Ace bandage or a sugar tong splint would be inappropriate because they do not immobilize the thumb. A long arm cast for 8 weeks would immobilize the thumb, but could lead to loss of function, and may overtreat the injury if it is not truly a scaphoid fracture.
-
Question 38 of 105
38. Question
An 18 year old female athlete has a history of constant groin pain for 1 week. There is limited hip motion on flexion and internal rotation of the right hip. Radiographs of the hip and pelvis are normal. The most likely diagnosis is
Correct
Stress fracture of the right femoral neck
Incorrect
Explanation:
Stress fractures of the femoral neck are most commonly seen in military recruits and runners. They present with persistent groin pain, and limited hip flexion and internal rotation. Radiographs may be normal early. Iliotibial band syndrome also occurs in runners and presents with stinging pain over the lateral femoral epicondyle. Osteitis pubis occurs in distance runners and presents with pain in the anterior pelvic area and tenderness over the symphysis pubis. Pelvic inflammatory disease is associated with abdominal pain and fever.
-
Question 39 of 105
39. Question
A 69-year-old woman has recently been operated on for a left hip replacement. All of the following are appropriate prophylaxis for deep-vein thrombosis EXCEPT
Correct
Aspirin
Incorrect
Explanation:
Aspirin has not been shown to affect the coagulation system and protect against pulmonary embolism and deep-vein thrombosis. It is used prophylactically for prevention of stroke and myocardial infarction. Warfarin would prolong the prothrombin time, and thus works as a vitamin K antagonist affecting coagulation factors 2, 7, 9, and 10. It is commonly used to prevent deep-vein thrombosis after orthopedic surgery. Pneumatic compression boots are mechanical devices that increase circulation by compressing the venous valves and preventing stasis, low molecular heparin is extremely safe and convenient to use to prevent deep-vein thrombosis after arthoplasty. Intravenous heparin affects the extrinsic pathway of the coagulation system and prolongs the partial prothrombin time. It is very short-acting and thus relatively safe postoperatively. If no contraindications occur, intravenous heparin can be used to prevent deep-vein thrombosis.
-
Question 40 of 105
40. Question
A 17-year-old male injures his foot by stepping on a nail that penetrates the sole of his tennis shoe and pierces the arch of his foot. He is given a tetanus shot and erythromycin. He presents again a week later with more pain, and radiography reveals a lytic lesion of the second metatarsal consistent with osteomyelitis. The likeliest responsible organism is
Correct
Pseudomonas aeruginosa.
Incorrect
Explanation:
Pseudomonas aeruginosa is the likeliest cause of osteomyelitis in the setting of a penetrating wound such as this. This organism tends to grow in moist places. Treatment with erythromycin would not affect Pseudomonas, but it might improve a staphylococcal infection. The other organisms would not cause a syndrome such as this.
-
Question 41 of 105
41. Question
A girl aged 17 year is brought to emergency room by her parents for severe right foot pain. The patient states that the pain started 1 day prior to presentation. She cannot recall any recent trauma, and denies any past medical or surgical problem. She is active and walks at least 1 hour daily in the nearby forest. She goes to high school and is doing very well. She gets along well with her parents except that they insist she should eat more as her weight has dropped from 130 to 105 pounds over the past year. The most likely diagnosis is
Correct
Metatarsal stress fracture
Incorrect
Explanation:
The patient is not eating well, as mentioned by her parents, and her weight loss (over 15% of baseline) as well as her school performance activity level is consistent with anorexia nervosa. Metatarsal stress fracture is a complication of rigorous prolonged walking or running in this population. While she is malnourished and underweight, she has poor insight and continues to decrease her input and increase her output with prolonged physical activity. The fracture is a complication of increased output beyond the patient´s physical limitation.
Conversion disorder (choice A) is manifested by chronic neurologic pain or deficit without any objective organic cause. This patient´s chief complaint is acute pain without anv sign of another neurologic deficit.
Diffuse muscle and joint aches in conjunction with lack of energy and reduced physical activity, is characteristic of depression (choice B). This patient´s pain is localized and she is very active. Ruptured or injured tendon of the ankle is a common injury, but the patient does not recall any recent injury and localization of the pain is not consistent with medial ankle injury (choice C). The girl´s daily walking in the nearby forest raises suspicion for tick bite (choice E) and Lyme disease with secondary joint pain. Arthritis associated with Lyme disease is generally centered around joints, and does not present as acutely as in this patient. -
Question 42 of 105
42. Question
Which one of the following is the most common bacterial cause of osteomyelitis?
Correct
Staphylococcus Aureus
Incorrect
Explanation:
Although Staphylococcus Aureus is the most common bacterial cause of osteomyelitis, its infection is often accompanied by Gram-negative rods and Gram-positive cocci, as well as anaerobes.
-
Question 43 of 105
43. Question
A 25-year-old white male presents with asymmetric arthritis of the left knee. X-ray reveals joint disease in the sacroiliac joints. Blood cultures for gonococci are negative. A diagnosis of Reiter´s syndrome is made. Which one of the following is LEAST likely to be associated with this condition?
Correct
Osteomyelitis
Incorrect
Explanation:
The tetrad findings of Reiter´s syndrome are arthritis, urethritis, conjunctivitis, and mucocutaneous lesions. Osteomyelitis is not associated with Reiter´s syndrome. Reiter´s syndrome is often preceded by enteric or venereal bacterial infection, but is not associated with gonococcal infections.
-
Question 44 of 105
44. Question
Osteoarthritis of the shoulders is most commonly found in
Correct
Ditch-diggers.
Incorrect
Explanation:
The shoulders are an unusual site for osteoarthritis, unless an individual has had chronic trauma, usually manifested by lifting or carrying heavy objects. Baseball pitchers and ditch-diggers have a higher incidence of shoulder osteoarthritis than others. There is no predilection for women or men. Long-distance runners and typists are not at increased risk.
-
Question 45 of 105
45. Question
A Colles´ fracture involves the
Correct
Radius.
Incorrect
Explanation:
Co1les´ fracture is a fracture of the radius.
-
Question 46 of 105
46. Question
A 41-year-old male who is status post-nephrectomy for renal clear cell carcinoma presents with a metastatic lesion to his tibia that requires surgical stabilization. Staging work-up does not reveal any other metastatic foci. Prior to surgery which one of the following interventions is most appropriate?
Correct
Angiographic embolization of the tumor
Incorrect
Explanation:
Renal clear cell carcinomas are highly vascular, and metastatic foci should be embolized prior to resection. Heparinization would place the patient at great risk of uncontrolled bleeding. Fresh frozen plasma would not be indicated unless coagulation factor abnormalities are present. There is no role for renal artery embolization. If preoperative antibiotics are chosen, one dose prior to incision is all that is recommended, and antibiotics are usually held until cultures are obtained.
-
Question 47 of 105
47. Question
The most important principle in the management of an acute compartment syndrome of the tibia is
Correct
Fasciotomy.
Incorrect
Explanation:
Acute compartment syndrome is a medical emergency requiring immediate surgical treatment, known as a fasciotomy, to allow the pressure to return to normal. Complete surgical decompression is essential in the management of any open fracture in order to prevent irreversible muscle necrosis. All of the remaining answer choices are important adjuvant treatments, but are worthless in the absence of adequate decompression.
-
Question 48 of 105
48. Question
Which one of the following is NOT true of osteomyelitis?
Correct
Involucrum develops if bacteria coat themselves and underlying surfaces with a protective biofilm.
Incorrect
Explanation:
Elevation of the periosteum results in deposition of new bone, known as involucrum, around the sequestrum. Bacteria protect themselves by adhering tightly to damaged bone and by coating themselves and the underlying surfaces with a protective biofilm. Trauma, ischemia, and foreign bodies all predispose to microbial invasion of bone. In an attempt to contain a bone infection, phagocytes release enzymes that lyse bone. In chronic infections, ischemic necrosis results in separation of large bony fragments known as sequestra. Subperiosteal, or soft tissue abscesses, form when pus breaks through the bone cortex. Staphylococcus aureus is responsible for 80% to 90% of the cases of pyogenic osteomyelitis in which an organism is recovered.
-
Question 49 of 105
49. Question
A 25 year old patient is brought after an automobile accident in which he suffered a posterior knee dislocation. The dislocation has been reduced and post-reduction X-rays show a good reduction and no fractures. The patient has a normal pulse and normal neurovascular exam. He is otherwise stable and no other injuries are present. Which of the following diagnostic studies should be ordered urgently?
Correct
Arteriogram
Incorrect
Explanation:
All patients with a history of posterior dislocations of the knee should have an arteriogram because of the high incidence of popliteal artery injury in these patients. An MRI (B) and stress radiographs (A) will reveal the extent of the ligamentous injury, but there is no urgency in obtaining these studies. If the patient had a dislocation, you already know that at least three of the four major knee ligaments are torn. A CT scan (C) would be useful to evaluate an occult tibia plateau fracture, but this is of secondary importance to any suspected arterial injury.
-
Question 50 of 105
50. Question
What is the initial management of a neurologically intact patient, seen in the emergency room with a fracture of the cervical spine?
Correct
Philadelphia collar.
Incorrect
Explanation:
All patients with known or suspected cervical spine fractures should be placed in a hard collar, such as the Philadelphia collar. Bedrest (A) and a soft collar (B) are inadequate immobilization of cervical spine fractures. Emergent surgery is not indicated (D) in a patient with a normal exam. While patients can be instructed to flex and extend their neck for X-ray the physician should never assist them (E), as this may worsen the injury.
-
Question 51 of 105
51. Question
A 16 year old boy has persistent pain in the right distal femur. The pain is not relieved by aspirin or other nonsteroidal inflammatory drugs. Radiologic studies demonstrate a mass that expands the distal femur and infiltrates the surrounding soft tissue. A biopsy is taken which shows a tumor composed of anaplastic cells and osteoid matrix. The limb must be amputated. The most appropriate diagnosis is
Correct
Osteosarcoma
Incorrect
Explanation:
The radiologic picture, gross features, patient´s age, and, histopathologic characteristics are consistent with osteosarcoma, a malignant mesenchymal neoplasm of bone origin. Osteosarcoma predominantly affects children and adolescents, but a second peak of incidence occurs in middle age. The later peak is in relation with known predisposing conditions, including Paget disease, bone infarcts, and ionizing radiation. Fifty percent of osteosarcomas in children arise around the knee or in the distal femur or proximal tibia. The most common clinical presentation is pain and/or a pathologic fracture. Histologically, the presence of a sarcoma that forms osteoid matrix is the defining feature. Osteoid matrix can be identified by its eosinophilic hyaline appearance on H&E sections.
Chondrosarcoma is a malignant mesenchymal tumor that develops in the pelvis, shoulders, or ribs of adults or elderly patients. Histologically, chondrosarcoma is characterized by formation of chondroid matrix, which appears slightly basophilic on H&E. Liposarcoma is a malignant mesenchymal tumor arising from the adipose tissue of retroperitoneum or thigh in middle-aged/elderly individuals. The defining histologic feature is the presence of lipoblasts, which are similar to immature precursors of adipocytes. Osteoid osteoma is a benign bone-forming tumor arising in adolescents and young adults. Intense bone pain that manifests at night and is relieved by aspirin is the most common presentation. It is composed of well differentiated bone trabeculae arranged haphazardly. Local excision is curative.
Rhabdomyosarcoma is a malignant neoplasm of skeletal muscle origin, occurring most commonly in children and young individuals. Neoplastic cells show characteristic cross striations and markers of skeletal muscle differentiation. -
Question 52 of 105
52. Question
A 62 year old woman undergoes bilateral hip replacement with removal of the proximal portions of both femurs for severe osteoarthritis. A bone sample is taken from the distal end of one of the removed specimens shows the changes of osteoarthritis and also the changes illustrated in the image. The changes present in the image are most suggestive of?
Correct
Osteoporosis
Incorrect
Explanation:
This woman has osteoporosis, which is common in postmenopausal women and causes a generalized decrease in bone mass with an increased risk of fracture. Microscopic features that suggest the diagnosis include thin, sparse trabeculae with normal calcification; and normal numbers and morphology of both osteoblasts and osteoclasts. The spaces between the trabeculae are not fibrosed. Microscopic features of hyperparathyroidism include increases in both osteoclasts and osteoblasts, decreased bony spicule size, fibrosis of marrow spaces with giant cells, and normal calcification of residual bone. Microscopic features of osteomalacia include normal sized trabeculae that are only partially calcified, “seams” of uncalcified osteoid, and often, increased osteoblasts. Microscopic features of Paget disease usually seen in its later stages include marked thickening of bony trabeculae with irregularly arranged cement lines (mosaic pattern). Unicameral bone cyst is an uncommon lesion usually diagnosed in children and young adults in which a long bone contains a cystic space filled with fluid and lined with connective tissue, granulation tissue collagen, and histiocytes.
-
Question 53 of 105
53. Question
A 52 year old female is brought to the emergency department after slipping and falling in her bathroom. She denies any weight loss but admits to weakness and constipation. Radiographs reveal a femoral neck fracture and also multiple sharply demarcated lytic bone lesions throughout her bones, including the skull. Biopsy of one of the lesions is performed and shown below. Based on the history and biopsy finding, further evaluation would most likely show
Correct
Monoclonal immunoglobulin spike
Incorrect
Explanation:
The photomicrograph shows a proliferation of abnormal plasma cells which is found in multiple myeloma. Other features of multiple myeloma can include lytic (often described as “punched-out”) bone lesions (which may undergo pathologic fracture), serum or urine monoclonal protein spikes, Bence Jones proteinuria, renal failure, anemia, and hyperviscosity syndrome. Abnormal spectrin is a feature of hereditary spherocytosis. Alpha fetoprotein is a tumor marker for hepatocellular carcinomas and testicular tumors. Excessive new bone growth with anterior bowing produces a pronounced anterior tibial convexity (saber shins). Saber shins are found in congenital syphilis. Vitamin D deficiency can cause osteomalacia and rickets but would not be associated with plasma cell proliferation.
-
Question 54 of 105
54. Question
A 44 year old man presents because of chronic fatigue. According to him he has stopped exercising, and feels tired after mild activity. A complete blood count reveals a hematocrit of 31, a white count of 1,950, and a platelet count of 90.000. The radiographic finding that would most likely lead to the correct diagnosis is?
Correct
Diffuse symmetric skeletal sclerosis
Incorrect
Explanation:
Trilineage failure of blood cell production, causing pancytopenia, suggests a generalized decrease in marrow cavity size. This can be due to bony overgrowth in processes such as osteopetrosis. This rare genetic disease is characterized by reduced osteoclast activity resulting in diffuse symmetric skeletal sclerosis. Despite the increased bone volume, the bones are abnormally brittle and fracture easily because osteoclast activity is essential for bone remodeling. The anemia observed in these patients is often very refractory to treatment. Osteomalacia produces a generalized decrease in bone mass, and is not associated with pancytopenia. A large calcified growth on the tibia suggests a benign or malignant tumor of bone, which would not usually replace enough marrow to cause anemia. Multiple exostoses would affect the outer surface, not the marrow cavity of the bone. A single, sharply demarcated (“punched out”) bony lesion located in the upper femur suggests Monostotic fibrous dysplasia, an asymptomatic, benign, bony lesion.
-
Question 55 of 105
55. Question
A 21 year old man develops a swollen and painful upper arm after a fall on basketball court. Over the next several weeks, the involved area becomes more circumscribed and firm and then later evolves to a painless, hard, well-demarcated mass. X-ray of the arm at this point would most likely show which of the following?
Correct
Flocculent radiodensities surrounding a radiolucent center
Incorrect
Explanation:
This is a typically presentation of myositis ossificans, which usually occur in athletic adolescents and young adults following trauma. Following muscle trauma, an area of damage heals with a fibroblastic proliferation that then ossifies, even though there is no connection to bone. Flocculent radiodensities surrounding a radiolucent center are seen on X-ray. The lesion can microscopically resemble extraskeletal osteosarcoma, but the clinical history is usually quite different (extraskeletal osteosarcoma is a disease of the elderly). Simple excision usually cures myositis ossificans.
-
Question 56 of 105
56. Question
A golfer accidentally hits the ground very hard, making a divot. His hand begins to hurt immediately, and he is taken to an emergency department. On physical examination marked tenderness of the palm near the little finger is found. Flexion of the little finger against resistance elicits pain at the same site. Which of the following is the most likely diagnosis?
Correct
Hook of hamate fracture
Incorrect
Explanation:
The hamate bone is the small bone in the wrist that sits directly proximal to the little and ring fingers. It has a bony protrusion, the hook of the hamate, which is very vulnerable to trauma of the palm, particularly in settings in which a person hits the ground forcibly with a stick (including a golf club) or on an outstretched hand. An X-ray film or CT scan of the carpal tunnel can confirm the diagnosis. Therapy involves open reduction with fixation or excision of the bone fragment. Cubital tunnel syndrome causes paresthesias and numbness in the ulnar side of the hand due to compression of the ulnar nerve at the elbow. Lunate dislocation is a displacement of the lunate with respect to the rest of the carpals. It can occur in severe trauma of the hand and is usually accompanied by compression of the median nerve, with resulting pain and numbness on the palmar aspect of the hand. Reflex sympathetic dystrophy typically causes pain and limited motion of the shoulder, accompanied by ipsilateral involvement of the hand. Scapholunate ligament rupture causes pain in the mid-wrist and usually occurs when a person falls onto an outstretched hand.
-
Question 57 of 105
57. Question
A 72 year old man puts some 8-ounce cans of vegetables on a shelf in his kitchen and develops the acute onset of severe right upper arm pain. He visits the local emergency room and a radiograph of his humerus shows a non displaced fracture. The most likely etiology for this lesion is:
Correct
Pathological fracture
Incorrect
Explanation:
This is a typical presentation for a pathological fracture through a bone infiltrated with a malignant process, such as metastatic tumor or a plasmacytoma. Such fractures occur in the absence of significant trauma. A rotator cuff tear, avulsion of the biceps, and polymyalgia rheumatica do not cause bone fractures. Osteogenesis imperfecta can cause fractures, but would be unlikely to present like this after minimal trauma.
-
Question 58 of 105
58. Question
Which of the following is the recommendation for patients that have undergone total hip replacement who plan to undergo dental procedures?
Correct
Oral amoxicillin prior to and after the procedure
Incorrect
Explanation:
The orthopaedic community currently follows the guidelines of the American Heart Association. There is less evidence supporting the use of prophylactic antibiotics in total joint arthroplasty patients than there is in heart valve patients.
-
Question 59 of 105
59. Question
A 66 year old woman slipped and fell on the ice. She was brought to the emergency department and X-rays were taken. She has no past medical history except osteoporosis. The fracture that her physician would most likely find is
Correct
Wrist fracture
Incorrect
Explanation:
Women 51-75 years of age are at risk for Type 1 osteoporosis, which is characterized by the loss of a greater amount of trabecular bone than cortical bone. The loss of trabecular bone mass puts the patient at risk for Colles´ fracture (fracture of the distal radius) and verterbral crush fractures. This woman is too young to be at significant risk for hip fracture, which is more common in patients with Type II (involutional or senile) osteoporosis, a condition that favors patients over the age of 70. Vertebral crush fractures due to osteoporosis I are common in women in this age group, but are more likely to result from weight bearing than a fall. Wrist fractures typically occur in the age group of 63-67. Skull fractures are rare in osteoporotic patients. Trabecular bone, found primarily in the central and axial Skelton, is affected in osteoporosis. The skull is considered cortical bone. Osteoporosis does not increase the risk of ankle fractures significantly Type I osteoporosis increases the risk of wrist and vertebral crush fractures; Type II increases the risk of fracture in the vertebrae, femoral neck, proximal humerus, proximal tibia, and pelvis.
-
Question 60 of 105
60. Question
Surgical emergency secondary to the increased risk of infection with delayed surgery occurs in which of the following?
Correct
Open tibia fracture
Incorrect
Explanation:
Open fractures should be irrigated and debrided immediately secondary to the increased risk of infection with delayed treatment. This risk is especially high in the tibia. The remainder choices have no increased risk of infection with delayed treatment.
-
Question 61 of 105
61. Question
Surgical urgency secondary to the increased risk of fat embolus with delayed surgery occurs in
Correct
Femoral shaft fracture
Incorrect
Explanation:
Femural shaft fractures should undergo prompt fixation secondary to the risk of fat embolism with delayed treatment. Only femur fractures have been shown to be consistently associated with this complication.
-
Question 62 of 105
62. Question
With delayed surgery high rate of neurovascular injury occurs in
Correct
Shoulder dislocation
Incorrect
Explanation:
An unreduced shoulder dislocation is associated with an increased incidence of axillary nerve injury. Ankle fractures, hip fractures, and forearm fractures do not have to be urgently fixed unless they are associated with an open injury or a dislocation. These fractures are not associated with fat embolism. Hip fracture can be considered an urgency in young patients, although this is secondary to the risk of avascular necrosis with delayed reduction.
-
Question 63 of 105
63. Question
Which one of the following is the best and safest current medical management of osteoarthritis?
Correct
Acetaminophen
Incorrect
Explanation:
Acetaminophen has been shown to be as effective as any nonsteroidal anti-inflammatory and is safer because of the lack of associated gastrointestinal bleeding attributed to long-term aspirin use. Prednisone is never indicated for the treatment of osteoarthritis. High-dose aspirin and indomethacin have not been shown to be any better than acetaminophen in this condition, and both have a much higher incidence of side effects than acetaminophen. Diclofenac is not more good than acetaminophen.
-
Question 64 of 105
64. Question
A 34 year old man sustained a work related injury to his left hand. He subsequently develops pain, swelling, loss of skin in the affected part. Radiography reveals osteoporosis of the hand. Which of the following is the most likely diagnosis?
Correct
Reflex sympathetic dystrophy.
Incorrect
Explanation:
Injury to a limb can lead to reflex sympathetic dystrophy. Osteosarcoma would present a different radiographic pattern. Septic arthritis is usually acute and associated with constitutional symptoms. Scleroderma is not related to injury and is usually symmetrical and associated with other features. Myxedema would also be symmetrical and associated with other symptoms. It, too, is not injury related.
-
Question 65 of 105
65. Question
A 66 year old woman presents to the emergency room with a pathologic fracture of the shaft of her humerus. Radiographic studies demonstrate multiple lytic and blastic bone lesions. Biopsy of one of these lesions shows adenocarcinoma. The most likely source of the primary tumor is which one of the following?
Correct
Breast
Incorrect
Explanation:
Breast cancer is unusual in that it produces both lytic and blastic metastases to bone. Breast and prostate cancers are the most common sources of bone metastases, but prostate metastases are usually blastic. Colon cancer does not usually metastasize to bone. Kidney cancer, lung cancer and thyroid cancer produce lytic lesions when they metastasize to bone.
-
Question 66 of 105
66. Question
A 70 year old male develops the insidious onset of pain in his pelvis. A random serum alkaline phosphatase level is found to be elevated. Correct diagnosis can be provided by which of the following?
Correct
Pelvic radiography
Incorrect
Explanation:
This patient´s symptoms and findings are most consistent with Paget´s disease, and radiography of the affected area is likely to show the lytic lesions and areas of increased bone density characteristic of this condition. Liver biopsy, hepatitis antibody screen, prostate biopsy, or bone biopsy are not indicated at this point.
-
Question 67 of 105
67. Question
A 6 year old girl is brought to the emergency room with a displaced distal radius fracture. Radial pulse is weak and the hand is cyanotic. Median nerve sensation is diminished. What should be the initial treatment?
Correct
Attempting to reduce the fracture.
Incorrect
Explanation:
The nerve and artery are most likely compressed by the fracture fragments. Initial treatment should be to reduce the fracture. If the neurovascular exam is normal after the reduction, the patient should be placed in a splint. Open reduction is only necessary if the neurovascular exam remains abnormal after closed reduction. Splinting, casting, and elevating are all part of the post reduction treatment, but are not appropriate until after a reduction and normal exam are obtained.
-
Question 68 of 105
68. Question
A 77 year old man has chronic osteoarthritis of the left knee. The orthopedic surgeon recommends total knee replacement. What treatment should be started after surgery to prevent the most common complication?
Correct
Sodium warfarin
Incorrect
Explanation:
The most common complication after total knee replacement is thromboembolic disease. Low dose warfarin is currently the most frequently used medication postoperatively to prevent deep vein thrombosis. Deep infections occur at a rate of approximately one percent over the life of knee replacements in patients with osteoarthritis. Thus, antibiotic prophylaxis is not needed. Debridement is done once a deep seated infection has been established in the knee joint. However, infection is not the most common complication of knee replacement surgery Aspirin is not adequate anticoagulation for post knee or post hip replacement. Aspirin can be used in combination with warfarin to prevent deep vein thrombosis. Physical therapy does increase mobilization, but it does not prevent pulmonary embolism secondary to deep vein thrombosis.
-
Question 69 of 105
69. Question
A 19 year old boy with a history of sickle cell disease falls and impacts a sharp stick deep into his upper arm, just above the elbow. He pulls the stick out and does not seek medical attention at that time. His arm heals but his leg starts to become sore over the next two months. He feels relatively ill and has fever. An X-ray film of his symptomatic leg is shown below. The most likely diagnosis is
Correct
Osteomyelitis
Incorrect
Explanation:
Osteomyelitis is a destructive inflammatory lesion of bone that can be caused by a wide variety of organisms, including aerobic and anaerobic bacteria, mycobacteria, and fungi. The infection may occur as a result of microorganism spread from adjacent infected tissues, contaminated fractures bone surgery, as well as hematogenous spread from a remote source as in this case. The most common site of hematologic seeding is the rapidly growing and highly vascular metaphysis of growing bones. Patients are usually febrile, and may additionally have weight loss and fatigue. Localized signs include warmth, swelling, erythema, and tenderness. Radiologic findings usually take 3 to 4 weeks to develop, and include bone destruction, soft tissue swelling, and periosteal elevation. Conditions that predispose to developing osteomyelitis include diabetes mellitus, sickle cell disease, AIDS, intravenous drug abuse, alcoholism, and immunosuppression. Ewing sarcoma does have a peak incidence between 10 and 20 years, and may cause either lytic destruction of bone or multiple layers of subperiosteal reactive bone formation. However, there would be no reason to suppose that a puncture wound would induce the tumor. While fracture is a clinical possibility with the scenario in the question, the X-ray film does not show either a fracture that is not healing or formation of the bony callus that would be seen with a healing fracture. Giant cell tumor most commonly occurs in persons in their 20s and 30s and tends to cause lytic lesions, visible on X-ray films that erode the epiphyses of a bone and produce soft tissue extensions. This patient´s history does not suggest the development of a tumor. Osteochondroma is the most common form of benign bone tumor, and occurs most often in persons aged 10 to 20. Osteochondroma has a distinctive radiologic appearance, since each osteochondroma is covered by a cartilaginous cap.
-
Question 70 of 105
70. Question
A 34 year old man sustains a compound femoral fracture after falling from ladder. He is recovering well after surgery until the third post operative day, when he develops acute shortness of breath. The best treatment for him is
Correct
Intravenous heparin
Incorrect
Explanation:
This patient most likely has a fat embolus as a result of his fracture and will best respond to intravenous heparin. The antiarrhythmic agent, lidocaine, would not be expected to help, nor would cardiac rehabilitation, or the antibiotic ciprofloxacin. While morphine sulfate might relieve some of his pain, it is only a palliative maneuver.
-
Question 71 of 105
71. Question
A 64 year old female decides to begin a rigorous exercise program, which includes long walks. She walks four miles on city streets on the first day. Next morning she awakens with sharp pain in the lateral aspect of the sole of her foot. The most likely cause of this discomfort is
Correct
Stress fracture of the fourth metatarsal bone.
Incorrect
Explanation:
This is sometimes called “march fracture,” since it was originally described in soldiers after long marches during wartime. This presentation is not consistent with any of the other conditions.
-
Question 72 of 105
72. Question
A 21 year old boy fractures his femoral head and several ribs and requires open reduction and internal fixation of his hip fracture and strapping of his chest wall. One week later he develops acute onset of palpitations. His ECG upon arrival to hospital is most likely to show
Correct
Atrial fibrillation.
Incorrect
Explanation:
Individuals who have had recent hip fractures and have restricted activity, are quite susceptible to pulmonary emboli, which can present with atrial fibrillation. Occasional ectopic beats would not produce acute palpitations serious enough to warrant an emergency trip to the hospital. An acute myocardial infarction would be very unlikely in an otherwise healthy 21 year old. Elevated ST segments would be expected with either a myocardial infarction or pericarditis, which is not likely in this individual. Similarly, ventricular flutter, another arrhythmia, would be unusual in an otherwise healthy young individual.
-
Question 73 of 105
73. Question
A 24 year old male fractures his femur in a skiing accident and is casted for six weeks. He is able to walk with little pain after removal of the cast. His radiograph is likely to show which one of the following?
Correct
Osteopenia.
Incorrect
Explanation:
Osteopenia is a common finding after prolonged disuse of a bone even in an otherwise healthy individual. This will resolve as the patient begins to walk. Non union of the fracture is not present because the patient can walk and has little pain. Ectopic bone formation would be very unusual in this setting. Loose bodies do not form around an old fracture. A lytic lesion would be very unusual in a healthy young person who can ambulate after his cast is removed.
-
Question 74 of 105
74. Question
Management of an open fracture should consist of which one of the following in addition to fracture reduction, fixation, and/or immobilization?
Correct
IV antibiotics, tetanus if not up to date, and emergent irrigation and debridement.
Incorrect
Explanation:
All patients with open fractures should be given intravenous antibiotics immediately, as well as tetanus, if this is not up to date. In addition, an open fracture is a surgical emergency and it should undergo irrigation and debridement within six hours of injury. Waiting for cultures will result in a greater chance of infection. While options A, B, and C are all part of the management of open fractures, proper management requires all three, and, thus, option E is the best answer.
-
Question 75 of 105
75. Question
A 33 year old woman underwent open arthrotomy to repair a torn anterior cruciate ligament in her right knee. She develops a painful nodule in the incision after a few months. Which of the following is represented by this nodule?
Correct
Neuroma.
Incorrect
Explanation:
A painful neuroma can develop in a surgical scar. The patient does not have rheumatold arthritis or gout. Fibrosarcomas can develop in scar tissues, but usually many years after the initial trauma. Lipomas are not usually painful and do not usually occur in scars.
-
Question 76 of 105
76. Question
A 73 year old female vacuums her floors. She notices severe mid thoracic pain the next morning that radiates around to her anterior chest. The chest X-ray is likely to show which one of the following as the cause of her pain?
Correct
A wedge shaped deformity of T8.
Incorrect
Explanation:
This is a typical presentation of a new vertebral compression fracture, which can radiate to the anterior chest. Cardiomegaly, pulmonary edema, aortic atherosclerosis, or degenerative disk disease would not be expected to present with these symptoms in this setting.
-
Question 77 of 105
77. Question
Which one of the following is NOT a surgical emergency?
Correct
A distal radius fracture in a five year old with obvious visual deformity and intra-articular displacement.
Incorrect
Explanation:
Orthopedic emergencies consist of those injuries with open wounds, neurovascular compromise, compartment syndrome, and irreducible dislocations. The patient described in does not meet any of these criteria. The patient with the swollen foot has signs and symptoms of a compartment syndrome. The patient with the tibia fracture has an open injury denoted by the location of the wound and fat in the drainage. The patient in choice C most likely has a popliteal artery injury, and the patient in choice D needs to undergo emergent reduction of the fracture to relieve pressure on the median nerve.
-
Question 78 of 105
78. Question
An 81 year old woman with severe osteoarthritis undergoes a left total knee replacement and subsequent physical therapy at home. Four weeks later she presents with fever and left leg pain. The most common complication postoperatively after total knee arthroplasty is
Correct
Deep vein thrombosis
Incorrect
Explanation:
The most common complication after total knee replacement is thromboembolic disease; the incidence of deep vein thrombosis is 25-50%. Warfarin prophylaxis is recommended. Aseptic loosening of knee implants is not common, less than 0.5% per year for cemented knee implants. Deep seated infections, primarily with Staph epidermidis or Staph aureus, occur at a rate of approximately 1% over the life of knee replacements. Osteoporosis would not present with fever and pain unless there was a fracture or infection. The patient would already have osteoporosis prior to the surgery. During total knee replacement, the anterior cruciate ligament is removed.
-
Question 79 of 105
79. Question
A 70 year old male has vertigo, tinnitus, and musculoskeletal pain. His ALP level was elevated to 1,000 mg/dl. Examination showed severe enlargement of the base of the skull and deformities of the long bones and clavicle. Which one of the following is NOT a feature of this disease?
Correct
X-rays are the most sensitive means to detect active lesions of this disease.
Incorrect
Explanation:
This patient has Paget´s disease of the bone. The bone scan, not X-rays, is the most sensitive means to detect active lesions of Paget´s disease. All of the remaining statements regarding Paget´s disease are true.
-
Question 80 of 105
80. Question
Which one of the following statement is FALSE regarding falls in the elderly?
Correct
More than 5% of falls result in a hip fracture.
Incorrect
Explanation:
Less than 5% of falls result in hip fractures. A low body mass index, thin body habitus, and low bone mineral density at the femoral neck can all predispose to a hip fracture after a fall. Osteoporosis is a major public health concern; it affects more than 25 million American men and women. The aim of primary prevention is to achieve optimal bone mass at maturity and maintain bone mass in adulthood so that fractures can be prevented after a fall. Prevention of falls can occur through exercise and rehabilitation programs. Exercise fitness programs and musculoskeletal rehabilitation programs can improve skeletal strength and decrease the risk of falling. Lack of estrogen in postmenopausal women, low body mass index, low bone mineral density and thinness can all contribute to an increased incidence of hip fractures after a fall. About 50% of fall injuries occur in and around the house. Biphosphonates, such as etidronate and alendronate, can decrease osteoclastic activity and, thus, decrease bone resorption. They have been shown to slow down the progression of osteoporosis along with estrogen, calcium, sodium fluoride, and nasal calcitonin. Optimal vitamin D intake maximizes the absorption of calcium from the gastrointestinal tract and, thus, appears effective in limiting bone loss.
-
Question 81 of 105
81. Question
A 60 year old female slips on the ice and sustains a left femur fracture. What is the most likely etiology of this?
Correct
Osteoporosis.
Incorrect
Explanation:
A 60 year old female is most likely to be postmenopausal and thus at risk for osteoporosis, which can increase the likelihood of bone fractures following trauma. Osteitis dessicans is usually a disease of youth and is not associated with trauma-induced femoral fractures. Osteoarthritis is not associated with increased risk of fractures. This is not a pathological fracture, since it is trauma induced, and, therefore, is much less likely to be the result of a metastatic lesion or a plasmacytoma.
-
Question 82 of 105
82. Question
A 63 year old man noticed that he cannot wear his hats properly because they no longer fit. Examination is remarkable for abnormal vertebral studies of L1-L4 with increased density in L2 and L3. Which of the following would be most likely to account for these changes?
Correct
Osteitis deformans
Incorrect
Explanation:
Vertebrae L2 and L3 have increased radiologic density, which suggests that there has been excessive bone deposition. The only condition listed in the answers that causes increases in bone density is osteitis deformans, also known as Paget disease. Paget disease of the bone has an unknown etiology, possibly related to a slow virus infection that stimulates bone metabolism. The process may involve one or many bones. Initially, bone resorption exceeds bone production, and so bone volume is reduced; but with time the osteoblastic process exceeds the osteoclastic one, and excess bone is laid down. Many patients are found incidentally, as was this one, when a casual remark suggesting a change in head size is mentioned to a physician. Other patients come to attention when they develop pathologic fractures or when bony ostia narrow and compress nerves or blood vessels. The most common neurologic symptom is hearing loss as a result of cochlear dysfunction and compression of the VIII cranial nerve. Fibrous dysplasia causes well circumscribed radiolucent areas on X-ray films. Hyperparathyroidism causes thinning of cortical bone with X-ray changes being most notable in the phalanges and in the mandible. Osteomalacia causes a generalized decreased density of bones, radiolucent bands known as pseudofractures, and bony deformity. Osteoporosis causes a generalized decrease in bony mass with cortical thinning and pathologic fractures.
-
Question 83 of 105
83. Question
A 75 year old diabetic woman undergoes a left total knee replacement for severe osteoarthritis. She takes daily NPH insulin and has good control over blood glucose. She also takes oxycodone for pain relief. She is planning on getting involved in a physical therapy rehabilitation program at the local hospital. On review of her medications, which one is most acutely indicated at this time?
Correct
Low molecular weight heparin
Incorrect
Explanation:
This patient is a post total knee replacement and is currently not on any anticoagulation therapy. The risk of deep venous thrombosis (DVT) and subsequent pulmonary embolism is very high in this population, and it is the standard of care to initiate an anticoagulant, such as low molecular weight heparin or warfarin. Low molecular weight heparin has proven more effective than unfractionated heparin for thromboprophylaxis following total knee replacement. A nonsteroidal anti-inflammatory agent (A) does not appear to be indicated at this time, as the patient appears to have reasonable pain control with her opiate. An ACE inhibitor (B) is, in the long term, an excellent drug for this patient given her diabetes. In the post surgical period, however, the greatest consideration should be given to the most pressing issue. Aspirin (C) is an anti-platelet agent that has no role in the prevention of DVT. Unfractionated heparin (E) is used for prevention of DVT in immobile patients or in hospitalized patients unable to ambulate. However, after orthopedic surgery, especially after joint procedures, its efficacy is very poor, given the increased venous stasis.
-
Question 84 of 105
84. Question
A 63 year old man presents to the emergency department with a swollen and extremely painful ankle. He had twisted his ankle while running to catch a bus and had fallen on his inverted foot. Examination shows swelling and angulation of the ankle. Anteroposterior (AP), lateral, and mortise X-ray films show displaced fractures of both malleoli. Which one would be the preferred form of treatment?
Correct
Open reduction and internal fixation
Incorrect
Explanation:
Precise alignment of the displaced fragments is needed to ensure that the tight mortise of the ankle joint is restored. Closed reduction and casting (A) is unlikely to provide the necessary realignment in a displaced lesion. It would, however, be an option in non-displaced injuries. Fusion of a joint (B) is the ultimate step when everything else has failed. It would not be the first choice for a relatively common fracture. Artificial joints are usually used for advanced articular disease. In the trauma setting, replacement with prosthesis (D) is as a rule reserved for fractures where avascular necrosis is predictable. Skeletal traction (E) in general, is indicated only in areas of the body where strong muscle groups pull broken bones into unacceptable positions, which is not the case here.
-
Question 85 of 105
85. Question
In the middle of a tennis game a 42 year old male falls to the ground clutching his ankle. He limps off the court with pain and swelling in the back of the lower leg. He presents the next day because of persistent pain, swelling, and limping. He can put weight on that foot with no exacerbation of the pain, but the motion of taking a step is painful. The likely finding on examination would be
Correct
There is absence of plantar flexion on squeezing of the gastrocnemius muscles at the prone position
Incorrect
Explanation:
The clinical description is that of a rupture of the Achilles tendon. The injured structure is so close to the skin that direct palpation of the gap in the tendon is usually possible. Inability of the prone positioned patient to plantar flex when the gastrocnemius muscles are squeezed indicates Achilles tendon rupture. This is also called a positive Thompson test. A fracture of the calcaneus (A) would happen with a fall from a height and landing on one´s feet. The ability to bend a joint beyond the normal boundaries (B and C) implies damage to the ligaments that keep that joint securely in place. However, such damage would not produce the loud popping noise so characteristic of rupture of the Achilles tendon. Grating and crepitation (E) are findings that indicate bony fracture; if these were present, the patient would not be able to put weight on the injured side.
-
Question 86 of 105
86. Question
A car is involved in an accident. The driver, who is wearing his seat belt, explains that he saw his drunk, unrestrained front seat passenger hit the windshield with his face, and the dashboard with his knees. Passenger´s exam shows multiple facial lacerations and shortened and adducted right lower extremity. Other joint examination is normal. He is neurologically intact and head CT is normal. A potential orthopedic emergency is not obvious but is suspected. Therefore, a radiologic image of which area should likely be ordered?
Correct
X-ray of both hips
Incorrect
Explanation:
When hitting the knees against the dashboard, the femurs can be driven backward and out of the acetabulum, resulting in posterior dislocation of the hips. Because of the tenuous blood supply of the femoral heads, such an injury must be promptly recognized and treated as it may cause avascular necrosis of the femoral head. Posterior dislocation of the hip can sometimes be recognized on physical examination as the affected lower extremity would be shortened, adducted, and internally rotated. Because there is high suspicion due to the nature of the injury and the physical finding, an X-ray of both hips should be ordered. MRI of both knees (A) should be done to look for posterior cruciate ligament tear, which is another possibility in such a dashboard injury, but it is not an emergency. Posterior dislocation of the knee is, in fact an orthopedic emergency because of the potential disruption of the popliteal artery, but there would be an obvious deformity would at the knee. Thus, this is not the best choice. The jaw (C) could indeed be fractured, but such fractures would be easily recognized clinically. If they were not identified until the next day, no damage would be incurred. Injury of the lumbar spine (D) should always be considered when someone falls from a height and lands on his feet, but it is not a likely hidden injury in this setting.
Skull X-ray films (E) have gone out of favor as a way to assess head injury. The main issue in head injuries is intracranial bleeding, and the study to confirm it is the CT scan. -
Question 87 of 105
87. Question
A 33 year old man is struck by a car going approximately 30mph. In the ED he is awake and talking and in no respiratory distress, but pulse is 110/min and BP is 136/68 mm Hg. Exam shows tenderness of the calf with significant edema and a thready right dorsalis pedis and posterior tibial pulse compared to the left lower extremity. X-rays confirm a tibial fracture. Next step in workup is
Correct
Angiography of the lower extremity
Incorrect
Explanation:
The patient has a closed fracture with weakened, but present, pulses. The fractures are not surgical emergencies, whereas a vascular injury might be. The key decision here is whether there is a definite or possible vascular injury. The ´hard´ signs of vascular injury to an extremity are managed with immediate surgical exploration in the operating room. These signs are (1) active hemorrhage (2) expanding hematoma (3) pulse deficit (3) bruit or thrill, and (4) distal ischemia. Distal ischemia is evident when the patient shows the classical signs of arterial insufficiency-the 6 P´s: paresthesia, poikilothermic, pulselessness, pallor, pain, and paralysis, although not necessarily in that order. Conversely, “soft” signs of vascular injury do not necessitate exploration, but imaging with an angiogram. (There is not yet sufficient evidence to support the use of CT angiography in this setting). “Soft” signs include (1) history of hemorrhage at the scene, (2) stable, non-expanding hematoma, (3) proximity to major vessel, (4) anatomically related nerve deficit, (5) ankle-brachial index of less than 0.9, and (6) unequal pulses. This patient has unequal pulses but not pulse deficit, classification his injury as possible and necessitating further workup with an angiogram (B). If the pulse were completely absent, immediate surgical, exploration (C) would be indicated. IV heparin (A) would be contraindicated in this trauma patient. His pulse change is likely due to vessel injury, not thrombosis or embolus. Observation (D) would be incorrect with the patient´s soil signs of a vascular injury and a known fracture, although the fracture is not open and does not, therefore, require emergent fixation (E). Diagnosis of a possible vascular injury takes precedence.
-
Question 88 of 105
88. Question
A 58 year old type 1 diabetic presents with left foot pain. He reports chronic pain on his left foot for about 6 months. He has poor dietary compliance takes his insulin as prescribed, but does not check his sugar regularly. His BP pressure is 164/88 mm Hg, heart rate is 95/min, and he is afebrile. He has 1+ palpable dorsalis pedis and posterior tibial pulses. On the distal aspect of the dorsum of his left foot, he has a clean 2 x 2 cm area of erythema that is tender but not fluctuant. He has bilateral loss of fine sensations. The most important intervention in his management is
Correct
Blood glucose control
Incorrect
Explanation:
The patient has diabetes mellitus which has been poorly controlled. He is showing early signs of complications, namely foot ulcers and loss of sensation. Management involves tight blood glucose control and frequent pediatric follow up examinations, as well as local wound care for open wounds. While the patient must also prevent other complications of diabetes, the question asks which intervention is most important in the management of the specific problem of his foot pain. With diabetes, hypertension, and a significant smoking history, he is at risk for cardiac and ophthalmologic complications, so an electrocardiogram (C) and ophthalmologic exam (D) would also be in order, as would risk reduction with smoking cessation (E), but that is not the answer to the question at hand. The other preventable factors for cardiac disease would be medical management of hypertension and diabetes. An angiogram (A) would not be indicated with palpable pulses, although this patient is at high risk for macrovascular disease and loss of pulses in the future if his risk factors are not modified.
-
Question 89 of 105
89. Question
A boy aged 8 years falls on his right hand with the arm extended, and he breaks his elbow by hyper-extension. X-ray shows a supracondylar fracture of the humerus. The fracture is reduced and immobilized. The mother asks if her son is going to be okay and what is “the worst that can happen.” Which complication is of greatest concern with this type of injury?
Correct
Vascular and nerve injuries
Incorrect
Explanation:
In general, fractures in children pose fewer problems than fractures in adults. However, this particular lesion in children is prone to result in vascular or nerve injuries and requires close follow-up immediately after it has been reduced and immobilized. Open reduction and internal fixation (A) are not necessarily the rule in these cases. Closed reduction is usually possible. Remodeling (B) is invariably excellent in fractures in children, and this one is no exception. Growth plate damage (C) occurs only when the fracture crosses the joint or when the growth plate is crushed, neither of which is the case here. Malunion (D) in the upper extremely is the big fear with displaced navicular fractures, but not with humeral fractures.
-
Question 90 of 105
90. Question
A 2-year-old boy after he fell in the bathtub has been limping and refusing to bear weight on his right leg. X ray shows a displaced transverse fracture of the femoral shaft without soft tissue changes. The child´s leg is immobilized and stabilized. CBC and coagulation studies are normal.
Which of the following is the most appropriate initial step in management?Correct
Obtain a skeletal survey
Incorrect
Explanation:
This patient has sustained a closed fracture of the femur shaft with no apparent bleeding into the thigh. Femoral shaft fractures result from high-energy trauma. In young children, one must always consider the possibility of abuse because approximately 70% of femur fractures are nonaccidental (i.e., inflicted). The patient is stable and immobilized and, at this point in management, requires a skeletal survey to identify other possible fractures and evaluate for potential abuse.
Application of traction closed reduction and external fixation and the placement of a spica cast are all ways of managing a fracture sustained in the femur. However, these constitute definitive management. The initial management is to complete a secondary survey and identify all possible injuries in the child with a high risk for abuse.
Obtaining a genetics consult for osteogenesis imperfecta is incorrect for the initial step of management. Osteogenesis imperfecta is a rare cause of pathologic fractures in children. Diagnosis requires extensive testing, and referral to a genetics consult would not constitute appropriate initial management before more common causes of femoral shaft fractures are evaluated.
Open reduction and internal fixation is performed in displaced epiphyseal fractures, in neurovascular injuries requiring repair, in open fractures of the femur and tibia, and following a failed attempt to obtain anatomic alignment. This child´s presentation is not an indication for surgery. Therefore, the most important initial step is to identify all injuries before definitive management. -
Question 91 of 105
91. Question
A 44 year old woman, who wears high-heeled, pointed shoes, has pain in the forefoot after prolonged standing or walking. She sometimes experiences numbness, a burning sensation, and tingling in the area. Exam shows a very tender spot in the third interspaced, between the third and fourth toes. There is no limitation of motion or signs of inflammation. The most likely diagnosis is
Correct
Morton´s neuroma
Incorrect
Explanation:
The location and circumstances are classic for Morton´s neuroma, a benign neuroma of the third plantar inter digital nerve. Gout (A) usually happens to obese, elderly males, and redness and signs of inflammation in the affected joint are evident. Hallux rigid us (B) is osteoarthritis of the first metatarsophalangeal joint. There is deformity and limitation of motion. The joint is tender on physical examination. Metatarsophalangeal articulation pain (C) is likewise associated with misalignment of joint surfaces. There is pain when examining the joint, and there is no history of numbness, burning, or tingling. Plantar fasciitis (E) produces sharp pain on physical examination when pressing the plantar surface of the heel.
-
Question 92 of 105
92. Question
A 30 year old footballer is referred for chronic knee pain. He has played football for over 10 years and cannot recall a time when his left knee did not hurt. The pain is significantly worse after extreme exertion. On exam, he has tenderness to palpation of the medial joint line and a trace effusion, but no limited range of motion or motor deficits. X-ray shows minimally narrowed joint space. MRI reveals a small tear in the medial meniscus but integrally of the all of the knee ligaments. The next appropriate step in the management is
Correct
Arthroscopic evaluation and mention
Incorrect
Explanation:
The diagnosis is given in this question and although the extent of the injury is not specified, the duration of symptoms is clear. Typically, management of maniacal injuries requires ruling out associated pathology (e.g., ligamentous injuries), which has been done in this case. Aggressive physical therapy (A) aimed at strengthening the quadriceps, normalizing gait, and encouraging a comfortable full range of motion is the. First line of treatment after an acute injury; this man´s injury, however, has been chronic and he is obviously physically active, so this would not likely cure his symptoms. NSAIDS (E) are useful for pain management and to continue with therapy but will not after the course of the pathology, especially in chronic conditions. Arthroscopic evaluation and intervention would be the correct answer since that allows for more accurate delineation of the existing pathology and allows for intervention (including debridement with partial meniscectomy), repair, or, rarely, maniacal transplantation. This is the treatment of option A after conservative management has faience. Open surgical intervention (D) is almost never indicated. Immobilization (C) is not indicated for maniacal injuries and would likely hurt the ambient by inducing muscle atrophy and not allowing for resolution of the effusion.
-
Question 93 of 105
93. Question
A 28 year old man presents with persistent right leg pain. He has “achy” pain at the hip radiating down his femur to the knee. It is worse on exertion but is also present at rest. It is usually relieved with NSAIDs. Medical history is significant for Crohn´s disease. His last flare-up was successfully managed with 2 weeks of prednisone orally. Examination is significant for limited range of the right hip, specifically with external rotation. X-ray shows a dulling of the femoral head. The next step in the diagnosis is
Correct
MRI
Incorrect
Explanation:
This patent´s diagnosis is avascular necrosis (AVN) of the femoral head secondary to exogenous steroid use. The classic symptoms of this condition include pain (often worse on external rotation), decreased range of motion, and sometimes a clicking with ambulation. The femoral head is particularly predisposed to a vascular necrosis secondary to its limited blood supply with little collateralization, specifically the end-artery lateral epiphysis vessels (originating from the medial fern oral circumflex artery form the profunda femoris). Exogenous steroid use is a well described cause of AVN, with the minimum dose of prednisone 20 mg daily for as brief time as 1-2 weeks, and can even cause AVN up to 3 years after discontinuation. The exact mechanism of this effect is unknown, but proposed theories include increased intraosseous pressure from hypertrophy of intramedullary fat cells, arterial occlusion due to fat emboli from the liver, inhibition of angiogenesis, and direct cellular toxicity to osteoclasts. Diagnosis can be made on plain X-ray alone in advanced cases; but early on, only a nonspecific flattening of the femoral head is visualized. Definitive diagnosis is with MRI, which carries a sensitively of 98% and a specificity of 85% before femoral head collapse, and sensitivity of 100% after femoral head collapse, significantly higher than any of the other diagnostic modalities offered as options. Additionally, MRI does not involve radiation exposure, which must be taken into consideration especially in young patients, as this disease entity exists in the pediatric population in the form of Legg-Calves-Perthes disease and slipped capital femoral epiphysis. None of the other choices will be able to visualize the bone to the extent of an MRI combined with the low amount of risk involved.
-
Question 94 of 105
94. Question
A 63 year old hypertensive is brought after a fall. He also has dyslipidemia, and migraines. X-ray confirms a femoral fracture that is corrected by open reduction and internal fixation are performed. For several post-op days he has limited range of motion, resulting in the need for bed rest and heavy narcotic use. What one is the appropriate for prevention of a DVT in him?
Correct
Low molecular weight heparin (LMWH) 40 mg subcutaneously daily
Incorrect
Explanation:
LMWH is fractionated heparin that has more specific binding potential than unfractionated heparin (E) and therefore has a more reliable effect on activated partial thromboplastin time. Additionally, it has the benefit of once daily dosing, easier on the patient and particularly useful in the outpatient setting. This patient is at high risk for a DVT given an orthopedic, lower extremity procedure. Therefore, prolonged bed rest is necessary due to extraordinary pain and narcotic use. Aggressive chemical prevention is definitely indicated, not just mechanical prevention with passive exercise (C) or the use sequential compression devices (D). Aspirin (A), through its anti-platelet effect, does prevent clotting to a degree, but the evidence to support its use over heparin products is limited, especially in such high-risk patient.
-
Question 95 of 105
95. Question
A 66 year old woman falls on her hand while getting off the chair. She presents with a deformed and painful wrist. Neurologic examination is normal. X-ray shows a dorsally displaced, dorsally angulated fracture of the distal radius and the ulna styloid. Which one is the most appropriate management?
Correct
Closed reduction, long arm cast
Incorrect
Explanation:
This is the famous Colles fracture, which typically can be reduced well with closed manipulation. Casting should immobilize both the wrist and the elbow to prevent any movement of the wrist; thus, a long arm cast is needed. A post-reduction radiograph is required for any closed reduction, but a short arm cast (A) would only partially immobilize the wrist, as some of the wrist-moving muscles (separators, predators) would remain mobile. An intramedullary fat rod (C) is usually reserved for fractures of the shaft of long bones, like the femur. Even though the fracture is displaced and angulated, open reduction and internal fixation (D) would be an unnecessarily expensive and intrusive way to do it. Closed reduction and long arm cast would serve the purpose for this case. Skeletal traction (E) could correct the deformity, but would do so at the cost of limiting future function of the hand.
-
Question 96 of 105
96. Question
A 28 year old man has pain and swelling in his right arm over the past 2 months that is worse when he raises his arm. Past history is unremarkable. He took up heavy weight lifting 1 year ago, and he uses several supplements for weight gain and muscle building. Exam shows a well built vitally stable male. Some pitting edema is noted in his right arm. He is instructed to raise his arm above his head. After 1 minute, the swelling increases markedly in his right arm. The most likely the source of his symptoms is
Correct
Anterior scalene muscle
Incorrect
Explanation:
The patient described has thoracic outlet syndrome (TOS), which is further subdivided into neurologic, arterial, or venous TOS. Neurologic symptoms result from impingement on the brachial plexus, which in turn causes paresthesia and muscular atrophy. Arterial occlusion leads to absent pulses and limb ischemia. Venous occlusion results in edema and venous engorgement. The patient´s symptoms most closely match venous TOS. Determining what is causing the obstruction is the next step in management. In this patient´s case, there is a history of significant weight lifting. In this context, hypertrophy of the scalene muscle can cause the obstruction. Treatment is aimed at relieving the obstruction surgically. Venous thrombosis is a complication of venous TOS, and directed thrombolysis can be performed after definitive surgical management. A cervical rib (B) can also cause thoracic outlet syndrome, and may manifest its symptoms due to blockade of neurovascular structure (brachial plexus and sub clavier vessels). Surgical resection of the rib relieves this obstruction. How ever this patient is experiencing thoracic outlet syndrome due to the anterior scalene muscle hypertrophy as indicated by his history or recent exercise change. Also a cervical rib would likely cause symptoms at an earlier age, as this is a congenital problem A pan coast tumor (C) would be reasonable in an older smoker, not in 28 year old healthy nonsmoker. Sub clavien steal syndrome (D) would present with vascular symptoms, manifesting as cold, tingling, and muscle pain, along with posterior neurological symptoms, such as visual problems, equilibrium disorders, and in extreme cases, syncope. The patient experiences the symptoms when the arm is exercised, or more commonly, syncope with muscular pain after trying to reach something above the head level. Sub clavien vein thrombosis (E) may be a complication of venous TOS; however, given the worsening of symptoms with abduction, anterior scalene hypertrophy is most likely causing the symptoms.
-
Question 97 of 105
97. Question
A 71 year old male is brought to the ED after a fall. He has coronary artery disease, chronic kidney disease, and osteoarthritis. On exam he has severe right hip tenderness, worse on abduction and external rotation, with significant soft tissue edema and ecchymosed, but no open wounds. The next step in his management is
Correct
X-ray of the right hip
Incorrect
Explanation:
The mechanism of injury and findings on physical exam are consistent with a fracture of the femur. This is also likely given the patient´s age and medical co-morbidities, making osteoporosis of some degree likely and raising his risk for fracture. Although CT (A) and MRI (B) can both be used to diagnose a femoral fracture, they are more expensive and usually unnecessary to make a diagnosis that can be made on plain X-ray (E).
Rarely, repair of a complex fracture may need preoperative planning with more advanced imaging (i.e. CT and MRI) modalities. An ultrasound of the deep venous system (C) would be the diagnostic test for deep venous thrombosis, but this diagnosis is incorrect. Most closed fractures do not need urgent operative repair (D), although there can be extenuating circumstances (e.g., a resultant compartment syndrome or hemodynamic instability due to blood loss). Open femoral fractures are at high risk for infection and should be repaired as soon as possible, usually within 6 hours of the incidence. -
Question 98 of 105
98. Question
According to the mother of 12 year old boy for the last 12 to 18 months he comes home and refused to exert himself in anyway because his knees hurt from physical education class at school. No motor or sensory deficits are noted in the lower limbs. Range of motion at all joints is normal; however, upon flexion of the knee joints he complains of pain in his proximal shin, and the knee joints are tender to palpation. Appropriate management is
Correct
Rest and ice after physical activity
Incorrect
Explanation:
The diagnosis in this case is Osgood-Shatter disease, which is the syndrome of pain in the tibia tubercle and patella after repetitive exercise. It occurs in children ages 10 to 15 years who are involved in sports, and it is usually worse with jumping. It occurs in this age group because of overuse of the quadriceps complex, whose distal insertion point is the patellar tendon into the tibia tubercle, which is still unclassified, making it prone to microscopic avulsion fractures and causing pain. Findings on physical exam include tenderness to palpation over the tibial tubercle and, in the acute phase, soft-tissue swelling and prominence of the tubercle. Range of motion is typically normal and no neurologic findings are present. The management of Osgood Shatter disease is conservative, mainly rest and icing of the painful area after exercise. Continuation of physical activity is not contraindicated and is actually encouraged. Operative intervention, usually excision of the tubercle (A) is rarely pursued and has not been shown to provide greater relief of symptoms. Although regular use of non-steroidal anti-inflammatory medications (D) may partially relieve some symptoms, these drugs do not alter the course of the condition and carry their own side effects, which are to be avoided in the adolescent population. The pathologic changes in this condition are extra articular, therefore intraarticular steroid injections (C) would be of little help. There is no role for long-term immobilization (B) or use of crutches in the management of Osgood-Shatter disease. Note that X-rays are not necessary for the diagnosis of Osgood-Shatter disease; but when obtained, X-rays may reveal gross fragmentation of the tibia tubercle.
-
Question 99 of 105
99. Question
A 13 year old boy injures his right shoulder and arm after a fall. In the ED he is noted to have extensive abrasions and ecchymoses over the lateral aspect of his right upper arm and shoulder, exquisite tenderness, and range of motion limited by pain. He does not complain of any distal parenthesis. Radial pulse is noted to be 2+. Radiographs reveal a distal humerus fracture. The orthopedic team reduces the fracture and casts the arm. While waiting for a repeat X-ray, he complains of numbness in his thumb, and he is found to be unable to extend his wrist. His radial pulse is still 2+. The next step in the management is
Correct
Re-manipulation by orthopedics
Incorrect
Explanation:
This patient´s diagnosis is radial nerve palsy secondary to the closed reduction preformed by the orthopedics team. The findings were not present at the initial presentation, implying that the initial manipulation is responsible for the symptoms described. The diagnosis of radial nerve palsy is based on the presence of both motor and sensory deficits along the radial nerve distribution, mainly extension of the wrist and sensation to the dorsal aspects of the lateral three and one-half digits. The radial nerve arises from the posterior cord of the brachial plexus with fibers from C6-T1. It innervates the extensor and separator muscles of the arm and forearm and provides sensation to the dorsal aspect of the radial three and one-half digits. It is of note that no intrinsic muscles of the hand are innervated by the radial nerve, nor is sensation to the distal forearm; therefore, both are spared in radial nerve injuries. Radial nerve palsy is most commonly caused by fracture of the humerus, especially in the middle third (Holstein-Lewis fracture) or at the junction of the middle and distal thirds. The nerve injury may occur acutely at the time of the injury, secondary to fracture manipulation, or chronically from a healing callus. In the present case the manipulation of the fracture has caused the palsy, and therefore the correct intervention should be re-manipulation to attempt to reduce the fracture without impinging on the nerve. Immediate operative exploration (A) following a closed fracture of the humerus is contraindicated due to the expected acute edema after the initial fracture and reduction. If the palsy developed from the initial fracture, it would not be incorrect to wait and see if the palsy improves as the postfracture edema resolves; however, opening such a closed wound would worsen the current situation in which the palsy was secondary to reduction and not to the fracture itself. If the palsy is not corrected with re-manipulation, then operative exploration would be correct finally, if radial nerve palsy developed during an open fracture, exploration of the course of the nerve and identification of any impingement would be indicated. The palsy is secondary to the manipulation and likely bony impingement not to muscle spasm; therefore, administering a muscle relaxant such as baclofen (B) would be incorrect. A repeat X-ray (D) is indicated to confirm adequate reduction; however, the bone will need to be re-manipulated and a repeat film will be obtained then. Therefore, a repeat X-ray now will not add any information and will detract time from treating the developing radial nerve palsy. Although warm compresses and elevation (E) would probably help with edema and possibly even relieve some of the compression of the nerve, they are not definitive treatment of a potentially disastrous complication.
-
Question 100 of 105
100. Question
A 59 year old policeman has intense, disabling, sharp heel pain every time his foot strikes the ground. It is worse in the mornings, preventing him from putting any weight on the heel. His BMI is 30 kg/m2. He shows exquisite tenderness to direct palpation over the anteromedial aspect of the heel. The pain is exacerbated when the toes are dorsiflexed. X-ray looks normal. The most likely diagnosis is
Correct
Planter fasciitis
Incorrect
Explanation:
The cause of plantar fasciitis may be multifactorial and is not completely understood. Accepted theories state that it might be caused by repetitive micro trauma. Possible risk factors include obesity, occupations requiring prolonged standing, heel spurs, peps planes, and reduced dorsiflexion of the ankle. In the past, the presence of bony spurs led many patients to undergo unnecessary surgery to remove the spur. The spur is caused by the pull of the fascia and is not the cause of the plantar fasciitis. Spontaneous resolution can be expected in 12 to 18 months, during which time symptomatic treatment is offered. Calcaneal stress fracture (A) would show up on the X-ray, which is normal in this case. Option C is not correct. In diagnosing ankle joint osteoarthritis, examination should reveal swelling of the joint, limited mobility, and pain with movement. In some cases, deformity, and/or enlargement (spur) of the joint may be noted. X-ray findings will show degenerative changes and narrowing of the joint space. Posterior Achilles tendon bursitis (E) occurs mostly in young women, not older men, and an erythematosus, indurate, tender area is present at the poster superior aspect of the heel.
-
Question 101 of 105
101. Question
An 11 year old boy is brought by his mother because he is not much physically active. He does not enjoy sports and is minimally active at physical education class in school. He admits that he is not active because the other children make fun of how he runs. On exam he is slim and noted to have bulkier gastrocnemius and soleus muscles on the left lower extremity and slightly limited abduction at the right hip. His gait evaluation reveals a persistently externally rotated femur. The most likely diagnosis is
Correct
Legg Calvé Perthes disease
Incorrect
Explanation:
Legg Calvé Perthes disease is a form of avascular necrosis of the femoral head of unknown etiology that presents in adolescents, males much more frequently than females. Presentation varies based on the degree of osteonecrosis, which usually correlates with radiographic findings. The disease is usually self-limited to 1-2 years and can be managed conservatively with observation and physical therapy. More symptomatic cases may need fixed hip abduction with casting; only the most severe cases requiring surgical intervention with an osteotomy. Osteoarthritis (C) is unlikely to be present in a patient of this age and would present as pain, not with this patient´s symptoms. Although osteosarcoma (A) present at this age, its symptom is usually pain, occasionally with a pathological fracture, neither of which are present in this case.
Polymyositis (D) is a skeletal muscle disorder that presents with muscle pain and cramping, especially on overexertion, but does not typically affect a single muscle group. Furthermore, it presents in an acute not sub-acute fashion, as in this patient´s condition. Although slipped capital femoral epiphysis (E) also presents in adolescent males as a gait abnormality, there is usually significant pain and the patients tend to be obese. -
Question 102 of 105
102. Question
A 36 year old man falls on his hand and presents with wrist pain. He was not able to break the fall, and that the heel of his hand took the brunt of his full weight as it hit the pavement. On exam he is distinctly tender to palpation over the anatomic snuff box. Anteroposterior and lateral X-rays are negative. The most likely diagnosis and most appropriate next step in management include
Correct
Carpal navicular fracture; thumb spice cast
Incorrect
Explanation:
Non displaced fractures of the carpal navicular are notorious for not showing up on X-ray films at the time of injury. The mechanism of injury plus the physical findings described in this vignette re sufficient to make a presumptive diagnosis and to indicate the use of a cast. Tenosynovitis (B) is not the result of a direct blow, but is seen in young mothers who carry the head of their babies with a hyper extended thumb and a flexed wrist. Displaced scaphoid fracture (C) would show up on X-ray films. The treatment for those fractures is indeed open reduction and internal fixation (they are notorious for non-healing), but the diagnosis is not correct in this option. Ligamentous injury (D) is often the assumption of those who are not aware of the peculiar nature of this injury. A similar mistake is made by those who assume twat if an X-ray film appeared negative, there cannot be a fracture (E)
-
Question 103 of 105
103. Question
A 54 year old woman falls and hurts her right shoulder. She presents to the ED with her arm held close to her body, but the forearm rotated outward as if she were going to shake hands. She is in pain and will not move the arm from that position. Her shoulder looks “square” in comparison with the rounded unhurt opposite side, and there is numbness in a small area of her shoulder over the deltoid muscle. The most likely diagnosis is
Correct
Anterior dislocation of the shoulder
Incorrect
Explanation:
Anterior dislocation of the shoulder is the most common dislocation of that joint. The position is classic, as is the lack of the rounded contour of the humeral head. The area of numbness represents injury to the auxiliary nerve, a common complication of anterior dislocation of the shoulder.
Acromioclavicular separation (A) would be characterized by very localized pain at that particular spot and none of the other features described here. Fracture of the humeral shaft (C) would likewise lack the specific deformity, inasmuch as the humeral head would still be in p ace to provide the normal rounded contour. Posterior dislocation (D) typically occurs following massive uncoordinated muscle contractions (electrical injuries, epileptic seizures), and the arm and forearm are held in a more “normal” protective position, close to the body.
Scapular fracture (E) happens only with extremely severe chest trauma; it would not happen by falling in the shower. Along with the axillary nervebe in pacewith two other injuries (fracture of the sternum or fracture of the first rib), scapular fracture, when present indicates that very severe trauma has occurred, and it is a useful clinical clue to look for hidden internal injuries. -
Question 104 of 105
104. Question
A 26 year old football player is brought to the ED after being rigorously tackled from behind during a game and is in severe left knee pain. On examination the left knee looks swollen and the leg can be pulled inferiorly with the knee flexed at 90 degrees. A similar finding can be elicited with the knee flexed at 20 other. Which one is the most appropriate management for this patient´s condition?
Correct
Anterior cruciate ligament surgical reconstruction
Incorrect
Explanation:
Swelling of the knee after trauma usually denotes the presence of a significant injury. The tests described (anterior drawer and Lachman test, respectively) are classic for an injury to the anterior cruciate ligament (ACL). Some studies suggest that women who practice jumping sports like volleyball, basketball, and football are more likely to develop ACL injuries than men. The next step in management of knee injuries is to obtain an MRI of the affected knee. Treatment could be either conservative or surgical. Athletes, such s the patient described above, require surgical reconstruction. (E) is inappropriate here, but would be appropriate for sedentary patients, who could be treated with knee brace immobilization and rehabilitation. Injury to the posterior cruciate ligament (PCL) (A) is unlikely here, since the drawer test described above indicates rupture of the anterior circulate ligament. The PCL is much less commonly injured than the ACL. When the PCL is injured, it produces opposite findings to those described above as if drawer were being closed rather than opened. The PCL injury is most likely associated with a blow in the shin coming from the front, which will displace the leg posterior (dashboard injury). Arthroscopic meniscectomy is not the priority here, since the lesion is clearly on the anterior cruciate ligament (C). Hinged cast for collateral ligament injury would be of some use if one of the collateral ligaments were injured (D). This injury would produce a leg that could be bent outward or inward more than the normal leg.
-
Question 105 of 105
105. Question
A 33 year old female notices pain in her right shoulder following a seizure and she cannot move it. She presents to a clinic, where she has a limited examination and negative AP X-rays of her shoulder. She is diagnosed as having a sprain and given pain medication. Next day, she still has the same pain and is unable to move her arm. She comes holding her arm close to the body, with her hand resting on her anterior chest wall. The likely diagnosis is
Correct
Posterior dislocation of the shoulder
Incorrect
Explanation:
The mechanism of injury (massive contraction of all muscles in the area) and the missed diagnosis on a single view AP X-ray film are classic for posterior dislocation of the shoulder. Axillary view X-ray films are needed to make the diagnosis. Acromioclavicular separation (A) would have been obvious on physical examination and on the X-ray film taken. Anterior dislocation (B) is far more common than posterior dislocation. However, it happens with regular trauma, has a very typical posture where the arm is held close to the body but the forearm and hand are rotated out as if (ready to shake hands), and is easily seen on X-ray films. Crushing of the articular cartilage (C) and tearing of shoulder girdle muscles (E) are not common injuries following seizures.